Location via proxy:   [ UP ]  
[Report a bug]   [Manage cookies]                

HTML conversions sometimes display errors due to content that did not convert correctly from the source. This paper uses the following packages that are not yet supported by the HTML conversion tool. Feedback on these issues are not necessary; they are known and are being worked on.

  • failed: beramono
  • failed: kpfonts
  • failed: shadethm

Authors: achieve the best HTML results from your LaTeX submissions by selecting from this list of supported packages.

License: CC BY-NC-SA 4.0
arXiv:2306.02745v2 [math-ph] 14 Dec 2023
Abstract:

We consider an abstract sequence {An}n=1superscriptsubscriptsubscript𝐴𝑛𝑛1\{A_{n}\}_{n=1}^{\infty}{ italic_A start_POSTSUBSCRIPT italic_n end_POSTSUBSCRIPT } start_POSTSUBSCRIPT italic_n = 1 end_POSTSUBSCRIPT start_POSTSUPERSCRIPT ∞ end_POSTSUPERSCRIPT of closed symmetric operators on a separable Hilbert space \mathcal{H}caligraphic_H. It is assumed that all Ansubscript𝐴𝑛A_{n}italic_A start_POSTSUBSCRIPT italic_n end_POSTSUBSCRIPT’s have equal deficiency indices (k,k)𝑘𝑘(k,k)( italic_k , italic_k ) and thus self-adjoint extensions {Bn}n=1superscriptsubscriptsubscript𝐵𝑛𝑛1\{B_{n}\}_{n=1}^{\infty}{ italic_B start_POSTSUBSCRIPT italic_n end_POSTSUBSCRIPT } start_POSTSUBSCRIPT italic_n = 1 end_POSTSUBSCRIPT start_POSTSUPERSCRIPT ∞ end_POSTSUPERSCRIPT exist and are parametrized by partial isometries {Un}n=1superscriptsubscriptsubscript𝑈𝑛𝑛1\{U_{n}\}_{n=1}^{\infty}{ italic_U start_POSTSUBSCRIPT italic_n end_POSTSUBSCRIPT } start_POSTSUBSCRIPT italic_n = 1 end_POSTSUBSCRIPT start_POSTSUPERSCRIPT ∞ end_POSTSUPERSCRIPT on \mathcal{H}caligraphic_H according to von Neumann’s extension theory. Under two different convergence assumptions on the Ansubscript𝐴𝑛A_{n}italic_A start_POSTSUBSCRIPT italic_n end_POSTSUBSCRIPT’s we give the precise connection between strong resolvent convergence of the Bnsubscript𝐵𝑛B_{n}italic_B start_POSTSUBSCRIPT italic_n end_POSTSUBSCRIPT’s and strong convergence of the Unsubscript𝑈𝑛U_{n}italic_U start_POSTSUBSCRIPT italic_n end_POSTSUBSCRIPT’s.

thm]Theorem thm]Lemma thm]Lemma thm]Proposition thm]Proposition thm]Corollary thm]Corollary thm]Definition thm]Definition

Convergence of operators with deficiency indices (k,k)𝑘𝑘(k,k)( italic_k , italic_k ) and of their self-adjoint extensions

August Bjergaabjerg@math.ku.dk

Department of Mathematical Sciences, University of Copenhagen,

Universitetsparken 5, DK-2100 Copenhagen Ø, Denmark

\abstractrunin

1 Introduction

We investigate in the following the notion of strong resolvent convergence of sequences of self-adjoint extensions of already specified (unbounded) closed symmetric operators on a Hilbert space. For the general theory on these topics we refer to [2] VIII and [1] X and introduce now the framework in which we will be working for the present section as well as for Section 3 where our main results are found. In Section 2 we treat also more general operators than considered here.

Consider a symmetric and closed operator A𝐴Aitalic_A on an infinite dimensional separable Hilbert space \mathcal{H}caligraphic_H111We adopt the convention that the inner product on \mathcal{H}caligraphic_H is linear in the second entry defined on a dense subspace D(A)𝐷𝐴D(A)italic_D ( italic_A ). The kernels :=Z(A*±i)assignsubscriptminus-or-plus𝑍plus-or-minussuperscript𝐴𝑖\mathcal{H}_{\mp}:=Z(A^{*}\pm i)caligraphic_H start_POSTSUBSCRIPT ∓ end_POSTSUBSCRIPT := italic_Z ( italic_A start_POSTSUPERSCRIPT * end_POSTSUPERSCRIPT ± italic_i ) are the deficiency subspaces and the pair (dim+,dim)dimensionsubscriptdimensionsubscript(\dim\mathcal{H}_{+},\dim\mathcal{H}_{-})( roman_dim caligraphic_H start_POSTSUBSCRIPT + end_POSTSUBSCRIPT , roman_dim caligraphic_H start_POSTSUBSCRIPT - end_POSTSUBSCRIPT ) is the deficiency indices. We assume that the latter are equal and finite, i.e. (dim+,dim)=(k,k)dimensionsubscriptdimensionsubscript𝑘𝑘(\dim\mathcal{H}_{+},\dim\mathcal{H}_{-})=(k,k)( roman_dim caligraphic_H start_POSTSUBSCRIPT + end_POSTSUBSCRIPT , roman_dim caligraphic_H start_POSTSUBSCRIPT - end_POSTSUBSCRIPT ) = ( italic_k , italic_k ) for some k=1,2,𝑘12k=1,2,\dotsitalic_k = 1 , 2 , … (however, see Remark Remark). This implies, cf. [1] Theorem X.2, that A𝐴Aitalic_A has self-adjoint extensions, and moreover any self-adjoint extension B𝐵Bitalic_B of A𝐴Aitalic_A is given by the rule

D(B)={ϕ0+ϕ++Uϕ+|ϕ0D(A),ϕ++},D(B)=\{\phi_{0}+\phi_{+}+U\phi_{+}\mathrel{|}\phi_{0}\in D(A),\;\phi_{+}\in% \mathcal{H}_{+}\},italic_D ( italic_B ) = { italic_ϕ start_POSTSUBSCRIPT 0 end_POSTSUBSCRIPT + italic_ϕ start_POSTSUBSCRIPT + end_POSTSUBSCRIPT + italic_U italic_ϕ start_POSTSUBSCRIPT + end_POSTSUBSCRIPT | italic_ϕ start_POSTSUBSCRIPT 0 end_POSTSUBSCRIPT ∈ italic_D ( italic_A ) , italic_ϕ start_POSTSUBSCRIPT + end_POSTSUBSCRIPT ∈ caligraphic_H start_POSTSUBSCRIPT + end_POSTSUBSCRIPT } ,
B(ϕ0+ϕ++Uϕ+)=Aϕ0+iϕ+iUϕ+𝐵subscriptitalic-ϕ0subscriptitalic-ϕ𝑈subscriptitalic-ϕ𝐴subscriptitalic-ϕ0𝑖subscriptitalic-ϕ𝑖𝑈subscriptitalic-ϕB(\phi_{0}+\phi_{+}+U\phi_{+})=A\phi_{0}+i\phi_{+}-iU\phi_{+}italic_B ( italic_ϕ start_POSTSUBSCRIPT 0 end_POSTSUBSCRIPT + italic_ϕ start_POSTSUBSCRIPT + end_POSTSUBSCRIPT + italic_U italic_ϕ start_POSTSUBSCRIPT + end_POSTSUBSCRIPT ) = italic_A italic_ϕ start_POSTSUBSCRIPT 0 end_POSTSUBSCRIPT + italic_i italic_ϕ start_POSTSUBSCRIPT + end_POSTSUBSCRIPT - italic_i italic_U italic_ϕ start_POSTSUBSCRIPT + end_POSTSUBSCRIPT

where U:+:𝑈subscriptsubscriptU\colon\mathcal{H}_{+}\to\mathcal{H}_{-}italic_U : caligraphic_H start_POSTSUBSCRIPT + end_POSTSUBSCRIPT → caligraphic_H start_POSTSUBSCRIPT - end_POSTSUBSCRIPT is a unitary map which can be extended to a partial isometry on all of \mathcal{H}caligraphic_H by letting Uϕ=0𝑈italic-ϕ0U\phi=0italic_U italic_ϕ = 0 for ϕ[+]italic-ϕsuperscriptdelimited-[]subscriptperpendicular-to\phi\in[\mathcal{H}_{+}]^{\perp}italic_ϕ ∈ [ caligraphic_H start_POSTSUBSCRIPT + end_POSTSUBSCRIPT ] start_POSTSUPERSCRIPT ⟂ end_POSTSUPERSCRIPT. Conversely, all extensions of A𝐴Aitalic_A of this form are self-adjoint.

We introduce now sequences {An}n=1superscriptsubscriptsubscript𝐴𝑛𝑛1\{A_{n}\}_{n=1}^{\infty}{ italic_A start_POSTSUBSCRIPT italic_n end_POSTSUBSCRIPT } start_POSTSUBSCRIPT italic_n = 1 end_POSTSUBSCRIPT start_POSTSUPERSCRIPT ∞ end_POSTSUPERSCRIPT and {Bn}n=1superscriptsubscriptsubscript𝐵𝑛𝑛1\{B_{n}\}_{n=1}^{\infty}{ italic_B start_POSTSUBSCRIPT italic_n end_POSTSUBSCRIPT } start_POSTSUBSCRIPT italic_n = 1 end_POSTSUBSCRIPT start_POSTSUPERSCRIPT ∞ end_POSTSUPERSCRIPT of such operators. That is, the Ansubscript𝐴𝑛A_{n}italic_A start_POSTSUBSCRIPT italic_n end_POSTSUBSCRIPT’s are densely defined, symmetric and closed operators on \mathcal{H}caligraphic_H with deficiency subspaces ±nsuperscriptsubscriptplus-or-minus𝑛\mathcal{H}_{\pm}^{n}caligraphic_H start_POSTSUBSCRIPT ± end_POSTSUBSCRIPT start_POSTSUPERSCRIPT italic_n end_POSTSUPERSCRIPT and deficiency indices (k,k)𝑘𝑘(k,k)( italic_k , italic_k ) independent of n𝑛nitalic_n, and Bnsubscript𝐵𝑛B_{n}italic_B start_POSTSUBSCRIPT italic_n end_POSTSUBSCRIPT is a self-adjoint extension of Ansubscript𝐴𝑛A_{n}italic_A start_POSTSUBSCRIPT italic_n end_POSTSUBSCRIPT defined by a unitary map Un:+nn:subscript𝑈𝑛superscriptsubscript𝑛superscriptsubscript𝑛U_{n}\colon\mathcal{H}_{+}^{n}\to\mathcal{H}_{-}^{n}italic_U start_POSTSUBSCRIPT italic_n end_POSTSUBSCRIPT : caligraphic_H start_POSTSUBSCRIPT + end_POSTSUBSCRIPT start_POSTSUPERSCRIPT italic_n end_POSTSUPERSCRIPT → caligraphic_H start_POSTSUBSCRIPT - end_POSTSUBSCRIPT start_POSTSUPERSCRIPT italic_n end_POSTSUPERSCRIPT (which can all, once again, be considered as partial isometries on \mathcal{H}caligraphic_H) as described above for each n𝑛nitalic_n. In this set-up we think of A𝐴Aitalic_A, B𝐵Bitalic_B and U𝑈Uitalic_U as limiting operators of the sequences of Ansubscript𝐴𝑛A_{n}italic_A start_POSTSUBSCRIPT italic_n end_POSTSUBSCRIPT’s, Bnsubscript𝐵𝑛B_{n}italic_B start_POSTSUBSCRIPT italic_n end_POSTSUBSCRIPT’s and Unsubscript𝑈𝑛U_{n}italic_U start_POSTSUBSCRIPT italic_n end_POSTSUBSCRIPT’s respectively, and our main goal will be to examine the interplay between the convergence of these sequences. A very natural question is for example whether we can obtain results along the lines of

"Suppose AnAnormal-→subscript𝐴𝑛𝐴A_{n}\to Aitalic_A start_POSTSUBSCRIPT italic_n end_POSTSUBSCRIPT → italic_A. Then BnBnormal-→subscript𝐵𝑛𝐵B_{n}\to Bitalic_B start_POSTSUBSCRIPT italic_n end_POSTSUBSCRIPT → italic_B if and only if UnUnormal-→subscript𝑈𝑛𝑈U_{n}\to Uitalic_U start_POSTSUBSCRIPT italic_n end_POSTSUBSCRIPT → italic_U." (1)

Of course one needs here to specify which notions of convergences we involve in this statement for it to be mathematically interesting. For the purposes of this note we focus on strong convergence of operators on Hilbert spaces. Hence, UnUsubscript𝑈𝑛𝑈U_{n}\to Uitalic_U start_POSTSUBSCRIPT italic_n end_POSTSUBSCRIPT → italic_U should be understood as usual strong convergence of bounded operators and BnBsubscript𝐵𝑛𝐵B_{n}\to Bitalic_B start_POSTSUBSCRIPT italic_n end_POSTSUBSCRIPT → italic_B as strong resolvent convergence of self-adjoint unbounded operators, i.e. as strong convergence of (Bn+i)1superscriptsubscript𝐵𝑛𝑖1(B_{n}+i)^{-1}( italic_B start_POSTSUBSCRIPT italic_n end_POSTSUBSCRIPT + italic_i ) start_POSTSUPERSCRIPT - 1 end_POSTSUPERSCRIPT towards (B+i)1superscript𝐵𝑖1(B+i)^{-1}( italic_B + italic_i ) start_POSTSUPERSCRIPT - 1 end_POSTSUPERSCRIPT – for an introduction to the topic and an explanation why this is in some sense the only "right" way of extending the concept of strong convergence to self-adjoint unbounded operators, see [2] VIII.7. For the Ansubscript𝐴𝑛A_{n}italic_A start_POSTSUBSCRIPT italic_n end_POSTSUBSCRIPT’s, however, we cannot use this generalized version of strong convergence since these are not self-adjoint.

This issue will be addressed in Section 2. Once this theoretical framework is in place, we will gradually progress towards presenting statements of the form (1) in Corollaries Corollary and Corollary. Finally, an exposition on the optimality of these results – in particular of the latter – is included for completeness.

Example \thethm.

As a final note before diving into technical details we mention the structure of a class of motivational examples that illuminates why we even care to search for results like (1).

Consider a sequence {A~n}n=1superscriptsubscriptsubscript~𝐴𝑛𝑛1\{\widetilde{A}_{n}\}_{n=1}^{\infty}{ over~ start_ARG italic_A end_ARG start_POSTSUBSCRIPT italic_n end_POSTSUBSCRIPT } start_POSTSUBSCRIPT italic_n = 1 end_POSTSUBSCRIPT start_POSTSUPERSCRIPT ∞ end_POSTSUPERSCRIPT of explicitly given symmetric differential operators on an open subset ΩΩ\Omegaroman_Ω of dsuperscript𝑑\mathbb{R}^{d}blackboard_R start_POSTSUPERSCRIPT italic_d end_POSTSUPERSCRIPT defined on D(A~n)=Cc(Ω)𝐷subscript~𝐴𝑛superscriptsubscript𝐶𝑐ΩD(\widetilde{A}_{n})=C_{c}^{\infty}(\Omega)italic_D ( over~ start_ARG italic_A end_ARG start_POSTSUBSCRIPT italic_n end_POSTSUBSCRIPT ) = italic_C start_POSTSUBSCRIPT italic_c end_POSTSUBSCRIPT start_POSTSUPERSCRIPT ∞ end_POSTSUPERSCRIPT ( roman_Ω ). Now the usual way to realize A~nsubscript~𝐴𝑛\widetilde{A}_{n}over~ start_ARG italic_A end_ARG start_POSTSUBSCRIPT italic_n end_POSTSUBSCRIPT as a self-adjoint operator on L2(Ω)=superscript𝐿2ΩL^{2}(\Omega)=\mathcal{H}italic_L start_POSTSUPERSCRIPT 2 end_POSTSUPERSCRIPT ( roman_Ω ) = caligraphic_H is the following: Let Ansubscript𝐴𝑛A_{n}italic_A start_POSTSUBSCRIPT italic_n end_POSTSUBSCRIPT be the closure of A~nsubscript~𝐴𝑛\widetilde{A}_{n}over~ start_ARG italic_A end_ARG start_POSTSUBSCRIPT italic_n end_POSTSUBSCRIPT for each n𝑛nitalic_n and if this is not already self-adjoint extend it by the above procedure to some self-adjoint operator Bnsubscript𝐵𝑛B_{n}italic_B start_POSTSUBSCRIPT italic_n end_POSTSUBSCRIPT. Here we have an example where the sequence {An}n=1superscriptsubscriptsubscript𝐴𝑛𝑛1\{A_{n}\}_{n=1}^{\infty}{ italic_A start_POSTSUBSCRIPT italic_n end_POSTSUBSCRIPT } start_POSTSUBSCRIPT italic_n = 1 end_POSTSUBSCRIPT start_POSTSUPERSCRIPT ∞ end_POSTSUPERSCRIPT is concretely described and not often subject to change. It describes not only how the Ansubscript𝐴𝑛A_{n}italic_A start_POSTSUBSCRIPT italic_n end_POSTSUBSCRIPT’s but also (through the An*superscriptsubscript𝐴𝑛A_{n}^{*}italic_A start_POSTSUBSCRIPT italic_n end_POSTSUBSCRIPT start_POSTSUPERSCRIPT * end_POSTSUPERSCRIPT’s) how the Bnsubscript𝐵𝑛B_{n}italic_B start_POSTSUBSCRIPT italic_n end_POSTSUBSCRIPT’s act on their domain, and often it will not be to difficult to prove that AnAsubscript𝐴𝑛𝐴A_{n}\to Aitalic_A start_POSTSUBSCRIPT italic_n end_POSTSUBSCRIPT → italic_A for some A𝐴Aitalic_A in an appropriate sense. We suppose that this convergence has been established. Moreover, natural examples of sequences of this form will in most cases satisfy the crucial property that all the operators have the same deficiency indices. The deficiency subspaces will be parts of solutions spaces of differential equations and usually the Unsubscript𝑈𝑛U_{n}italic_U start_POSTSUBSCRIPT italic_n end_POSTSUBSCRIPT’s will be simple maps between such spaces. Hence, in this case, strong convergence of the Unsubscript𝑈𝑛U_{n}italic_U start_POSTSUBSCRIPT italic_n end_POSTSUBSCRIPT’s is a property which is a lot easier to handle than the full strong resolvent convergence of the Bnsubscript𝐵𝑛B_{n}italic_B start_POSTSUBSCRIPT italic_n end_POSTSUBSCRIPT’s.

Now one can envision a couple of situations: If a sequence of Bnsubscript𝐵𝑛B_{n}italic_B start_POSTSUBSCRIPT italic_n end_POSTSUBSCRIPT’s is known, (1) could help us determine a self-adjoint extension B𝐵Bitalic_B of A𝐴Aitalic_A so that BnBsubscript𝐵𝑛𝐵B_{n}\to Bitalic_B start_POSTSUBSCRIPT italic_n end_POSTSUBSCRIPT → italic_B in the strong resolvent sense. One needs only to find the strong limit of the Unsubscript𝑈𝑛U_{n}italic_U start_POSTSUBSCRIPT italic_n end_POSTSUBSCRIPT’s (if this exists) and use this to extend A𝐴Aitalic_A. If the strong limit of the Unsubscript𝑈𝑛U_{n}italic_U start_POSTSUBSCRIPT italic_n end_POSTSUBSCRIPT’s does not exists then the result will conversely tell us that the Bnsubscript𝐵𝑛B_{n}italic_B start_POSTSUBSCRIPT italic_n end_POSTSUBSCRIPT’s do not converge towards any self-adjoint extension of A𝐴Aitalic_A. On the other hand it could be that B𝐵Bitalic_B was a fixed self-adjoint extension of A𝐴Aitalic_A and the result could in the same manner be used to find a sequence of Bnsubscript𝐵𝑛B_{n}italic_B start_POSTSUBSCRIPT italic_n end_POSTSUBSCRIPT’s which extends the Ansubscript𝐴𝑛A_{n}italic_A start_POSTSUBSCRIPT italic_n end_POSTSUBSCRIPT’s and converge towards B𝐵Bitalic_B in the strong resolvent sense – or whether such sequence exists at all. \scriptstyle\blacktriangle

2 Strong graph convergence and convergence of graph projections

Now some candidates for types of convergences for the Ansubscript𝐴𝑛A_{n}italic_A start_POSTSUBSCRIPT italic_n end_POSTSUBSCRIPT’s in (1) are treated. Along the way we introduce the machinery needed for both formulating and proving our main results. Firstly we need to introduce a particular notion of convergence of subspaces of a Hilbert space.

Definition.

Let {Vn}n=1superscriptsubscriptsubscript𝑉𝑛𝑛1\{V_{n}\}_{n=1}^{\infty}{ italic_V start_POSTSUBSCRIPT italic_n end_POSTSUBSCRIPT } start_POSTSUBSCRIPT italic_n = 1 end_POSTSUBSCRIPT start_POSTSUPERSCRIPT ∞ end_POSTSUPERSCRIPT be a sequence of subspaces of a Hilbert space \mathcal{H}caligraphic_H. The subspace

V:={x|There exists a sequence {xn}n=1 withxnVn for each n so that xnx as n}assignsubscript𝑉𝑥|There exists a sequence {xn}n=1 withxnVn for each n so that xnx as nV_{\infty}:=\Big{\{}x\in\mathcal{H}\mathrel{\Big{|}}\begin{subarray}{c}\text{% There exists a sequence $\{x_{n}\}_{n=1}^{\infty}\subseteq\mathcal{H}$ with}\\ \text{$x_{n}\in V_{n}$ for each $n$ so that $x_{n}\to x$ as $n\to\infty$}\end{% subarray}\Big{\}}italic_V start_POSTSUBSCRIPT ∞ end_POSTSUBSCRIPT := { italic_x ∈ caligraphic_H | start_ARG start_ROW start_CELL There exists a sequence { italic_x start_POSTSUBSCRIPT italic_n end_POSTSUBSCRIPT } start_POSTSUBSCRIPT italic_n = 1 end_POSTSUBSCRIPT start_POSTSUPERSCRIPT ∞ end_POSTSUPERSCRIPT ⊆ caligraphic_H with end_CELL end_ROW start_ROW start_CELL italic_x start_POSTSUBSCRIPT italic_n end_POSTSUBSCRIPT ∈ italic_V start_POSTSUBSCRIPT italic_n end_POSTSUBSCRIPT for each italic_n so that italic_x start_POSTSUBSCRIPT italic_n end_POSTSUBSCRIPT → italic_x as italic_n → ∞ end_CELL end_ROW end_ARG }

is called the strong limit of {Vn}n=1superscriptsubscriptsubscript𝑉𝑛𝑛1\{V_{n}\}_{n=1}^{\infty}{ italic_V start_POSTSUBSCRIPT italic_n end_POSTSUBSCRIPT } start_POSTSUBSCRIPT italic_n = 1 end_POSTSUBSCRIPT start_POSTSUPERSCRIPT ∞ end_POSTSUPERSCRIPT and we write VnVsubscript𝑉𝑛subscript𝑉V_{n}\to V_{\infty}italic_V start_POSTSUBSCRIPT italic_n end_POSTSUBSCRIPT → italic_V start_POSTSUBSCRIPT ∞ end_POSTSUBSCRIPT strongly. \square

One should not be misled by the fact that we call this type of convergence "strong". We note that any sequence of subspaces has a limit in the above sense (although it might be the trivial 00-subspace), and hence this way of converging cannot be a particularly strong one. The word "strong" merely refers to the fact that {xn}n=1superscriptsubscriptsubscript𝑥𝑛𝑛1\{x_{n}\}_{n=1}^{\infty}{ italic_x start_POSTSUBSCRIPT italic_n end_POSTSUBSCRIPT } start_POSTSUBSCRIPT italic_n = 1 end_POSTSUBSCRIPT start_POSTSUPERSCRIPT ∞ end_POSTSUPERSCRIPT should converge towards x𝑥xitalic_x strongly, i.e. with respect to the Hilbert space norm.

Another notion of convergence of sequences of closed subspaces of a Hilbert space is that of the orthogonal projections onto these converging strongly towards the orthogonal projection onto a limiting subspace. In fact, this is generally a stronger notion of convergence of subspaces than the above "strong" convergence.

Lemma.

Let {Vn}n=1superscriptsubscriptsubscript𝑉𝑛𝑛1\{V_{n}\}_{n=1}^{\infty}{ italic_V start_POSTSUBSCRIPT italic_n end_POSTSUBSCRIPT } start_POSTSUBSCRIPT italic_n = 1 end_POSTSUBSCRIPT start_POSTSUPERSCRIPT ∞ end_POSTSUPERSCRIPT be a sequence of closed subspaces of a Hilbert space \mathcal{H}caligraphic_H and denote the orthogonal projections onto these by {Pn}n=1superscriptsubscriptsubscript𝑃𝑛𝑛1\{P_{n}\}_{n=1}^{\infty}{ italic_P start_POSTSUBSCRIPT italic_n end_POSTSUBSCRIPT } start_POSTSUBSCRIPT italic_n = 1 end_POSTSUBSCRIPT start_POSTSUPERSCRIPT ∞ end_POSTSUPERSCRIPT. Denote similarly by P𝑃Pitalic_P the orthogonal projection onto another subspace V𝑉V\subseteq\mathcal{H}italic_V ⊆ caligraphic_H.

  • (a)

    V𝑉Vitalic_V is contained in the strong limit of {Vn}n=1superscriptsubscriptsubscript𝑉𝑛𝑛1\{V_{n}\}_{n=1}^{\infty}{ italic_V start_POSTSUBSCRIPT italic_n end_POSTSUBSCRIPT } start_POSTSUBSCRIPT italic_n = 1 end_POSTSUBSCRIPT start_POSTSUPERSCRIPT ∞ end_POSTSUPERSCRIPT if and only if Pnxx=Pxsubscript𝑃𝑛𝑥𝑥𝑃𝑥P_{n}x\to x=Pxitalic_P start_POSTSUBSCRIPT italic_n end_POSTSUBSCRIPT italic_x → italic_x = italic_P italic_x for all xV𝑥𝑉x\in Vitalic_x ∈ italic_V.

  • (b)

    If PnPsubscript𝑃𝑛𝑃P_{n}\to Pitalic_P start_POSTSUBSCRIPT italic_n end_POSTSUBSCRIPT → italic_P strongly then V𝑉Vitalic_V is the strong limit of {Vn}n=1superscriptsubscriptsubscript𝑉𝑛𝑛1\{V_{n}\}_{n=1}^{\infty}{ italic_V start_POSTSUBSCRIPT italic_n end_POSTSUBSCRIPT } start_POSTSUBSCRIPT italic_n = 1 end_POSTSUBSCRIPT start_POSTSUPERSCRIPT ∞ end_POSTSUPERSCRIPT.

\square

Proof.

(a): Assume on the one hand that V𝑉Vitalic_V is contained in the strong limit of {Vn}n=1superscriptsubscriptsubscript𝑉𝑛𝑛1\{V_{n}\}_{n=1}^{\infty}{ italic_V start_POSTSUBSCRIPT italic_n end_POSTSUBSCRIPT } start_POSTSUBSCRIPT italic_n = 1 end_POSTSUBSCRIPT start_POSTSUPERSCRIPT ∞ end_POSTSUPERSCRIPT. Then, for any xV𝑥𝑉x\in Vitalic_x ∈ italic_V, there exists a sequence {xn}n=1superscriptsubscriptsubscript𝑥𝑛𝑛1\{x_{n}\}_{n=1}^{\infty}\subseteq\mathcal{H}{ italic_x start_POSTSUBSCRIPT italic_n end_POSTSUBSCRIPT } start_POSTSUBSCRIPT italic_n = 1 end_POSTSUBSCRIPT start_POSTSUPERSCRIPT ∞ end_POSTSUPERSCRIPT ⊆ caligraphic_H with xnVnsubscript𝑥𝑛subscript𝑉𝑛x_{n}\in V_{n}italic_x start_POSTSUBSCRIPT italic_n end_POSTSUBSCRIPT ∈ italic_V start_POSTSUBSCRIPT italic_n end_POSTSUBSCRIPT for all n𝑛nitalic_n so that xnxsubscript𝑥𝑛𝑥x_{n}\to xitalic_x start_POSTSUBSCRIPT italic_n end_POSTSUBSCRIPT → italic_x. Hence, Pnxxxnx0delimited-∥∥subscript𝑃𝑛𝑥𝑥delimited-∥∥subscript𝑥𝑛𝑥0\lVert P_{n}x-x\rVert\leq\lVert x_{n}-x\rVert\longrightarrow 0∥ italic_P start_POSTSUBSCRIPT italic_n end_POSTSUBSCRIPT italic_x - italic_x ∥ ≤ ∥ italic_x start_POSTSUBSCRIPT italic_n end_POSTSUBSCRIPT - italic_x ∥ ⟶ 0 as needed. The other implication is clear if one considers the sequence {Pnx}n=1superscriptsubscriptsubscript𝑃𝑛𝑥𝑛1\{P_{n}x\}_{n=1}^{\infty}{ italic_P start_POSTSUBSCRIPT italic_n end_POSTSUBSCRIPT italic_x } start_POSTSUBSCRIPT italic_n = 1 end_POSTSUBSCRIPT start_POSTSUPERSCRIPT ∞ end_POSTSUPERSCRIPT for each xV𝑥𝑉x\in Vitalic_x ∈ italic_V.

(b): Assume PnPsubscript𝑃𝑛𝑃P_{n}\to Pitalic_P start_POSTSUBSCRIPT italic_n end_POSTSUBSCRIPT → italic_P strongly and denote by Vsubscript𝑉V_{\infty}italic_V start_POSTSUBSCRIPT ∞ end_POSTSUBSCRIPT the strong limit of {Vn}n=1superscriptsubscriptsubscript𝑉𝑛𝑛1\{V_{n}\}_{n=1}^{\infty}{ italic_V start_POSTSUBSCRIPT italic_n end_POSTSUBSCRIPT } start_POSTSUBSCRIPT italic_n = 1 end_POSTSUBSCRIPT start_POSTSUPERSCRIPT ∞ end_POSTSUPERSCRIPT. By (a) we need only to argue that VVsubscript𝑉𝑉V_{\infty}\subseteq Vitalic_V start_POSTSUBSCRIPT ∞ end_POSTSUBSCRIPT ⊆ italic_V or equivalently VVsuperscript𝑉perpendicular-tosuperscriptsubscript𝑉perpendicular-toV^{\perp}\subseteq V_{\infty}^{\perp}italic_V start_POSTSUPERSCRIPT ⟂ end_POSTSUPERSCRIPT ⊆ italic_V start_POSTSUBSCRIPT ∞ end_POSTSUBSCRIPT start_POSTSUPERSCRIPT ⟂ end_POSTSUPERSCRIPT. However, if yV𝑦superscript𝑉perpendicular-toy\in V^{\perp}italic_y ∈ italic_V start_POSTSUPERSCRIPT ⟂ end_POSTSUPERSCRIPT then for any xV𝑥subscript𝑉x\in V_{\infty}italic_x ∈ italic_V start_POSTSUBSCRIPT ∞ end_POSTSUBSCRIPT we can choose a sequence {xn}n=1superscriptsubscriptsubscript𝑥𝑛𝑛1\{x_{n}\}_{n=1}^{\infty}\subseteq\mathcal{H}{ italic_x start_POSTSUBSCRIPT italic_n end_POSTSUBSCRIPT } start_POSTSUBSCRIPT italic_n = 1 end_POSTSUBSCRIPT start_POSTSUPERSCRIPT ∞ end_POSTSUPERSCRIPT ⊆ caligraphic_H as for the x𝑥xitalic_x in (a) and obtain

y,x=limn(1Pn)y,xn=0𝑦𝑥subscript𝑛1subscript𝑃𝑛𝑦subscript𝑥𝑛0\langle y,x\rangle=\lim_{n\to\infty}\langle(1-P_{n})y,x_{n}\rangle=0⟨ italic_y , italic_x ⟩ = roman_lim start_POSTSUBSCRIPT italic_n → ∞ end_POSTSUBSCRIPT ⟨ ( 1 - italic_P start_POSTSUBSCRIPT italic_n end_POSTSUBSCRIPT ) italic_y , italic_x start_POSTSUBSCRIPT italic_n end_POSTSUBSCRIPT ⟩ = 0

proving yV𝑦superscriptsubscript𝑉perpendicular-toy\in V_{\infty}^{\perp}italic_y ∈ italic_V start_POSTSUBSCRIPT ∞ end_POSTSUBSCRIPT start_POSTSUPERSCRIPT ⟂ end_POSTSUPERSCRIPT as needed. \square

Remark.

While Lemma Lemma(b) shows that convergence of projections is a stronger type of convergence than "strong" convergence in the sense of Definition Definition, the following example shows that it is actually strictly stronger – a fact which will be important later on.

Consider a sequence {Vn}n=1superscriptsubscriptsubscript𝑉𝑛𝑛1\{V_{n}\}_{n=1}^{\infty}{ italic_V start_POSTSUBSCRIPT italic_n end_POSTSUBSCRIPT } start_POSTSUBSCRIPT italic_n = 1 end_POSTSUBSCRIPT start_POSTSUPERSCRIPT ∞ end_POSTSUPERSCRIPT of subspaces of a Hilbert space \mathcal{H}caligraphic_H of the form Vn=[xn]subscript𝑉𝑛superscriptdelimited-[]subscript𝑥𝑛perpendicular-toV_{n}=[\mathbb{C}x_{n}]^{\perp}italic_V start_POSTSUBSCRIPT italic_n end_POSTSUBSCRIPT = [ blackboard_C italic_x start_POSTSUBSCRIPT italic_n end_POSTSUBSCRIPT ] start_POSTSUPERSCRIPT ⟂ end_POSTSUPERSCRIPT where xnsubscript𝑥𝑛x_{n}\in\mathcal{H}italic_x start_POSTSUBSCRIPT italic_n end_POSTSUBSCRIPT ∈ caligraphic_H is of unit length and denote by Vsubscript𝑉V_{\infty}italic_V start_POSTSUBSCRIPT ∞ end_POSTSUBSCRIPT the strong limit of this sequence. Suppose that xn=x0subscript𝑥𝑛subscript𝑥0x_{n}=x_{0}italic_x start_POSTSUBSCRIPT italic_n end_POSTSUBSCRIPT = italic_x start_POSTSUBSCRIPT 0 end_POSTSUBSCRIPT is fixed for n𝑛nitalic_n odd and xn=ynsubscript𝑥𝑛subscript𝑦𝑛x_{n}=y_{n}italic_x start_POSTSUBSCRIPT italic_n end_POSTSUBSCRIPT = italic_y start_POSTSUBSCRIPT italic_n end_POSTSUBSCRIPT for n𝑛nitalic_n even where {yn}n=1superscriptsubscriptsubscript𝑦𝑛𝑛1\{y_{n}\}_{n=1}^{\infty}{ italic_y start_POSTSUBSCRIPT italic_n end_POSTSUBSCRIPT } start_POSTSUBSCRIPT italic_n = 1 end_POSTSUBSCRIPT start_POSTSUPERSCRIPT ∞ end_POSTSUPERSCRIPT is a sequence which is weekly convergent towards 00. Now x0Vsubscript𝑥0subscript𝑉x_{0}\notin V_{\infty}italic_x start_POSTSUBSCRIPT 0 end_POSTSUBSCRIPT ∉ italic_V start_POSTSUBSCRIPT ∞ end_POSTSUBSCRIPT since for n𝑛nitalic_n odd we have dist(Vn,x0)=1distsubscript𝑉𝑛subscript𝑥01\operatorname{dist}(V_{n},x_{0})=1roman_dist ( italic_V start_POSTSUBSCRIPT italic_n end_POSTSUBSCRIPT , italic_x start_POSTSUBSCRIPT 0 end_POSTSUBSCRIPT ) = 1. If, however, x[x0]𝑥superscriptdelimited-[]subscript𝑥0perpendicular-tox\in[\mathbb{C}x_{0}]^{\perp}italic_x ∈ [ blackboard_C italic_x start_POSTSUBSCRIPT 0 end_POSTSUBSCRIPT ] start_POSTSUPERSCRIPT ⟂ end_POSTSUPERSCRIPT then we can consider the sequence znsubscript𝑧𝑛z_{n}italic_z start_POSTSUBSCRIPT italic_n end_POSTSUBSCRIPT which is xVn𝑥subscript𝑉𝑛x\in V_{n}italic_x ∈ italic_V start_POSTSUBSCRIPT italic_n end_POSTSUBSCRIPT for n𝑛nitalic_n odd and xyn,xynVn𝑥subscript𝑦𝑛𝑥subscript𝑦𝑛subscript𝑉𝑛x-\langle y_{n},x\rangle y_{n}\in V_{n}italic_x - ⟨ italic_y start_POSTSUBSCRIPT italic_n end_POSTSUBSCRIPT , italic_x ⟩ italic_y start_POSTSUBSCRIPT italic_n end_POSTSUBSCRIPT ∈ italic_V start_POSTSUBSCRIPT italic_n end_POSTSUBSCRIPT for n𝑛nitalic_n even. As yn,x0subscript𝑦𝑛𝑥0\langle y_{n},x\rangle\to 0⟨ italic_y start_POSTSUBSCRIPT italic_n end_POSTSUBSCRIPT , italic_x ⟩ → 0 we see that znxsubscript𝑧𝑛𝑥z_{n}\to xitalic_z start_POSTSUBSCRIPT italic_n end_POSTSUBSCRIPT → italic_x proving xV𝑥subscript𝑉x\in V_{\infty}italic_x ∈ italic_V start_POSTSUBSCRIPT ∞ end_POSTSUBSCRIPT. We conclude that V=[x0]subscript𝑉superscriptdelimited-[]subscript𝑥0perpendicular-toV_{\infty}=[\mathbb{C}x_{0}]^{\perp}italic_V start_POSTSUBSCRIPT ∞ end_POSTSUBSCRIPT = [ blackboard_C italic_x start_POSTSUBSCRIPT 0 end_POSTSUBSCRIPT ] start_POSTSUPERSCRIPT ⟂ end_POSTSUPERSCRIPT.

On the the other hand the orthogonal projections Pnsubscript𝑃𝑛P_{n}italic_P start_POSTSUBSCRIPT italic_n end_POSTSUBSCRIPT onto the Vnsubscript𝑉𝑛V_{n}italic_V start_POSTSUBSCRIPT italic_n end_POSTSUBSCRIPT’s do not converge strongly at all. In particular Pnx0subscript𝑃𝑛subscript𝑥0P_{n}x_{0}italic_P start_POSTSUBSCRIPT italic_n end_POSTSUBSCRIPT italic_x start_POSTSUBSCRIPT 0 end_POSTSUBSCRIPT is 00 for n𝑛nitalic_n odd and x0yn,x0ynx0subscript𝑥0subscript𝑦𝑛subscript𝑥0subscript𝑦𝑛subscript𝑥0x_{0}-\langle y_{n},x_{0}\rangle y_{n}\to x_{0}italic_x start_POSTSUBSCRIPT 0 end_POSTSUBSCRIPT - ⟨ italic_y start_POSTSUBSCRIPT italic_n end_POSTSUBSCRIPT , italic_x start_POSTSUBSCRIPT 0 end_POSTSUBSCRIPT ⟩ italic_y start_POSTSUBSCRIPT italic_n end_POSTSUBSCRIPT → italic_x start_POSTSUBSCRIPT 0 end_POSTSUBSCRIPT for n𝑛nitalic_n even. \square

Letting operators once again enter the picture we can now easily define a notion of convergence of any sequence of operators on a Hilbert space: The strong graph convergence which is also treated in [2] VIII.7.

Definition.

Let {An}n=1superscriptsubscriptsubscript𝐴𝑛𝑛1\{A_{n}\}_{n=1}^{\infty}{ italic_A start_POSTSUBSCRIPT italic_n end_POSTSUBSCRIPT } start_POSTSUBSCRIPT italic_n = 1 end_POSTSUBSCRIPT start_POSTSUPERSCRIPT ∞ end_POSTSUPERSCRIPT be any sequence of operators on a fixed Hilbert space \mathcal{H}caligraphic_H. If the graphs Gr(An)Grsubscript𝐴𝑛\operatorname{Gr}(A_{n})roman_Gr ( italic_A start_POSTSUBSCRIPT italic_n end_POSTSUBSCRIPT ) converge strongly towards the graph Gr(A)Gr𝐴\operatorname{Gr}(A)roman_Gr ( italic_A ) of some operator A𝐴Aitalic_A on \mathcal{H}caligraphic_H as subspaces of direct-sum\mathcal{H}\oplus\mathcal{H}caligraphic_H ⊕ caligraphic_H then we say that A𝐴Aitalic_A is the strong graph limit of the Ansubscript𝐴𝑛A_{n}italic_A start_POSTSUBSCRIPT italic_n end_POSTSUBSCRIPT’s and write A=str.gr.limAn𝐴formulae-sequencestrgrlimsubscript𝐴𝑛A=\operatorname{str.gr.lim}A_{n}italic_A = start_OPFUNCTION roman_str . roman_gr . roman_lim end_OPFUNCTION italic_A start_POSTSUBSCRIPT italic_n end_POSTSUBSCRIPT. \square

Let us return to the case of a sequence of densely defined and closed operators {An}n=1superscriptsubscriptsubscript𝐴𝑛𝑛1\{A_{n}\}_{n=1}^{\infty}{ italic_A start_POSTSUBSCRIPT italic_n end_POSTSUBSCRIPT } start_POSTSUBSCRIPT italic_n = 1 end_POSTSUBSCRIPT start_POSTSUPERSCRIPT ∞ end_POSTSUPERSCRIPT for the remaining part of the section and fix once and for all the following convenient notation: By ΓsubscriptΓ\Gamma_{\infty}roman_Γ start_POSTSUBSCRIPT ∞ end_POSTSUBSCRIPT we mean the strong limit of {Gr(An)}n=1superscriptsubscriptGrsubscript𝐴𝑛𝑛1\{\operatorname{Gr}(A_{n})\}_{n=1}^{\infty}{ roman_Gr ( italic_A start_POSTSUBSCRIPT italic_n end_POSTSUBSCRIPT ) } start_POSTSUBSCRIPT italic_n = 1 end_POSTSUBSCRIPT start_POSTSUPERSCRIPT ∞ end_POSTSUPERSCRIPT and by Γ*superscriptsubscriptΓ\Gamma_{\infty}^{*}roman_Γ start_POSTSUBSCRIPT ∞ end_POSTSUBSCRIPT start_POSTSUPERSCRIPT * end_POSTSUPERSCRIPT the strong limit of {Gr(An*)}n=1superscriptsubscriptGrsuperscriptsubscript𝐴𝑛𝑛1\{\operatorname{Gr}(A_{n}^{*})\}_{n=1}^{\infty}{ roman_Gr ( italic_A start_POSTSUBSCRIPT italic_n end_POSTSUBSCRIPT start_POSTSUPERSCRIPT * end_POSTSUPERSCRIPT ) } start_POSTSUBSCRIPT italic_n = 1 end_POSTSUBSCRIPT start_POSTSUPERSCRIPT ∞ end_POSTSUPERSCRIPT. Note that (ϕ,ψ)Γitalic-ϕ𝜓subscriptΓ(\phi,\psi)\in\Gamma_{\infty}( italic_ϕ , italic_ψ ) ∈ roman_Γ start_POSTSUBSCRIPT ∞ end_POSTSUBSCRIPT if and only if there exists a sequence {ϕn}n=1superscriptsubscriptsubscriptitalic-ϕ𝑛𝑛1\{\phi_{n}\}_{n=1}^{\infty}\subseteq\mathcal{H}{ italic_ϕ start_POSTSUBSCRIPT italic_n end_POSTSUBSCRIPT } start_POSTSUBSCRIPT italic_n = 1 end_POSTSUBSCRIPT start_POSTSUPERSCRIPT ∞ end_POSTSUPERSCRIPT ⊆ caligraphic_H such that both ϕnϕsubscriptitalic-ϕ𝑛italic-ϕ\phi_{n}\to\phiitalic_ϕ start_POSTSUBSCRIPT italic_n end_POSTSUBSCRIPT → italic_ϕ and Anϕnψsubscript𝐴𝑛subscriptitalic-ϕ𝑛𝜓A_{n}\phi_{n}\to\psiitalic_A start_POSTSUBSCRIPT italic_n end_POSTSUBSCRIPT italic_ϕ start_POSTSUBSCRIPT italic_n end_POSTSUBSCRIPT → italic_ψ, and we have the similar characterization of Γ*superscriptsubscriptΓ\Gamma_{\infty}^{*}roman_Γ start_POSTSUBSCRIPT ∞ end_POSTSUBSCRIPT start_POSTSUPERSCRIPT * end_POSTSUPERSCRIPT. We can now present some basic properties of these subspaces.

Lemma.

Let {An}n=1superscriptsubscriptsubscript𝐴𝑛𝑛1\{A_{n}\}_{n=1}^{\infty}{ italic_A start_POSTSUBSCRIPT italic_n end_POSTSUBSCRIPT } start_POSTSUBSCRIPT italic_n = 1 end_POSTSUBSCRIPT start_POSTSUPERSCRIPT ∞ end_POSTSUPERSCRIPT be a sequence of densely defined and closed operators and let A𝐴Aitalic_A be an operator with the same properties as the Ansubscript𝐴𝑛A_{n}italic_A start_POSTSUBSCRIPT italic_n end_POSTSUBSCRIPT’s.

  • (a)

    If Gr(A)ΓGr𝐴subscriptΓ\operatorname{Gr}(A)\subseteq\Gamma_{\infty}roman_Gr ( italic_A ) ⊆ roman_Γ start_POSTSUBSCRIPT ∞ end_POSTSUBSCRIPT then Γ*Gr(A*)superscriptsubscriptΓGrsuperscript𝐴\Gamma_{\infty}^{*}\subseteq\operatorname{Gr}(A^{*})roman_Γ start_POSTSUBSCRIPT ∞ end_POSTSUBSCRIPT start_POSTSUPERSCRIPT * end_POSTSUPERSCRIPT ⊆ roman_Gr ( italic_A start_POSTSUPERSCRIPT * end_POSTSUPERSCRIPT ).

  • (b)

    If Gr(A)ΓGr𝐴subscriptΓ\operatorname{Gr}(A)\subseteq\Gamma_{\infty}roman_Gr ( italic_A ) ⊆ roman_Γ start_POSTSUBSCRIPT ∞ end_POSTSUBSCRIPT and Gr(A*)Γ*Grsuperscript𝐴superscriptsubscriptΓ\operatorname{Gr}(A^{*})\subseteq\Gamma_{\infty}^{*}roman_Gr ( italic_A start_POSTSUPERSCRIPT * end_POSTSUPERSCRIPT ) ⊆ roman_Γ start_POSTSUBSCRIPT ∞ end_POSTSUBSCRIPT start_POSTSUPERSCRIPT * end_POSTSUPERSCRIPT then Gr(A)=ΓGr𝐴subscriptΓ\operatorname{Gr}(A)=\Gamma_{\infty}roman_Gr ( italic_A ) = roman_Γ start_POSTSUBSCRIPT ∞ end_POSTSUBSCRIPT and Gr(A*)=Γ*Grsuperscript𝐴superscriptsubscriptΓ\operatorname{Gr}(A^{*})=\Gamma_{\infty}^{*}roman_Gr ( italic_A start_POSTSUPERSCRIPT * end_POSTSUPERSCRIPT ) = roman_Γ start_POSTSUBSCRIPT ∞ end_POSTSUBSCRIPT start_POSTSUPERSCRIPT * end_POSTSUPERSCRIPT

  • (c)

    If moreover the Ansubscript𝐴𝑛A_{n}italic_A start_POSTSUBSCRIPT italic_n end_POSTSUBSCRIPT’s are symmetric and A𝐴Aitalic_A is self-adjoint then A=str.gr.limAn𝐴formulae-sequencestrgrlimsubscript𝐴𝑛A=\operatorname{str.gr.lim}A_{n}italic_A = start_OPFUNCTION roman_str . roman_gr . roman_lim end_OPFUNCTION italic_A start_POSTSUBSCRIPT italic_n end_POSTSUBSCRIPT if and only if Gr(A)ΓGr𝐴subscriptΓ\operatorname{Gr}(A)\subseteq\Gamma_{\infty}roman_Gr ( italic_A ) ⊆ roman_Γ start_POSTSUBSCRIPT ∞ end_POSTSUBSCRIPT.

\square

Proof.

(a): Take (ϕ,ψ)Γ*italic-ϕ𝜓superscriptsubscriptΓ(\phi,\psi)\in\Gamma_{\infty}^{*}( italic_ϕ , italic_ψ ) ∈ roman_Γ start_POSTSUBSCRIPT ∞ end_POSTSUBSCRIPT start_POSTSUPERSCRIPT * end_POSTSUPERSCRIPT arbitrary and a corresponding sequence {ϕn}n=1superscriptsubscriptsubscriptitalic-ϕ𝑛𝑛1\{\phi_{n}\}_{n=1}^{\infty}{ italic_ϕ start_POSTSUBSCRIPT italic_n end_POSTSUBSCRIPT } start_POSTSUBSCRIPT italic_n = 1 end_POSTSUBSCRIPT start_POSTSUPERSCRIPT ∞ end_POSTSUPERSCRIPT with ϕnD(An*)subscriptitalic-ϕ𝑛𝐷superscriptsubscript𝐴𝑛\phi_{n}\in D(A_{n}^{*})italic_ϕ start_POSTSUBSCRIPT italic_n end_POSTSUBSCRIPT ∈ italic_D ( italic_A start_POSTSUBSCRIPT italic_n end_POSTSUBSCRIPT start_POSTSUPERSCRIPT * end_POSTSUPERSCRIPT ) so that ϕnϕsubscriptitalic-ϕ𝑛italic-ϕ\phi_{n}\to\phiitalic_ϕ start_POSTSUBSCRIPT italic_n end_POSTSUBSCRIPT → italic_ϕ and An*ϕnψsuperscriptsubscript𝐴𝑛subscriptitalic-ϕ𝑛𝜓A_{n}^{*}\phi_{n}\to\psiitalic_A start_POSTSUBSCRIPT italic_n end_POSTSUBSCRIPT start_POSTSUPERSCRIPT * end_POSTSUPERSCRIPT italic_ϕ start_POSTSUBSCRIPT italic_n end_POSTSUBSCRIPT → italic_ψ. Now for any ηD(A)𝜂𝐷𝐴\eta\in D(A)italic_η ∈ italic_D ( italic_A ) there exists a sequence {ηn}n=1superscriptsubscriptsubscript𝜂𝑛𝑛1\{\eta_{n}\}_{n=1}^{\infty}{ italic_η start_POSTSUBSCRIPT italic_n end_POSTSUBSCRIPT } start_POSTSUBSCRIPT italic_n = 1 end_POSTSUBSCRIPT start_POSTSUPERSCRIPT ∞ end_POSTSUPERSCRIPT with ηnD(An)subscript𝜂𝑛𝐷subscript𝐴𝑛\eta_{n}\in D(A_{n})italic_η start_POSTSUBSCRIPT italic_n end_POSTSUBSCRIPT ∈ italic_D ( italic_A start_POSTSUBSCRIPT italic_n end_POSTSUBSCRIPT ) so that ηnηsubscript𝜂𝑛𝜂\eta_{n}\to\etaitalic_η start_POSTSUBSCRIPT italic_n end_POSTSUBSCRIPT → italic_η and AnηnAηsubscript𝐴𝑛subscript𝜂𝑛𝐴𝜂A_{n}\eta_{n}\to A\etaitalic_A start_POSTSUBSCRIPT italic_n end_POSTSUBSCRIPT italic_η start_POSTSUBSCRIPT italic_n end_POSTSUBSCRIPT → italic_A italic_η. Using these sequences we see that

ϕ,Aη=limnϕn,Anηn=limnAn*ϕn,ηn=ψ,ηitalic-ϕ𝐴𝜂subscript𝑛subscriptitalic-ϕ𝑛subscript𝐴𝑛subscript𝜂𝑛subscript𝑛superscriptsubscript𝐴𝑛subscriptitalic-ϕ𝑛subscript𝜂𝑛𝜓𝜂\langle\phi,A\eta\rangle=\lim_{n\to\infty}\langle\phi_{n},A_{n}\eta_{n}\rangle% =\lim_{n\to\infty}\langle A_{n}^{*}\phi_{n},\eta_{n}\rangle=\langle\psi,\eta\rangle⟨ italic_ϕ , italic_A italic_η ⟩ = roman_lim start_POSTSUBSCRIPT italic_n → ∞ end_POSTSUBSCRIPT ⟨ italic_ϕ start_POSTSUBSCRIPT italic_n end_POSTSUBSCRIPT , italic_A start_POSTSUBSCRIPT italic_n end_POSTSUBSCRIPT italic_η start_POSTSUBSCRIPT italic_n end_POSTSUBSCRIPT ⟩ = roman_lim start_POSTSUBSCRIPT italic_n → ∞ end_POSTSUBSCRIPT ⟨ italic_A start_POSTSUBSCRIPT italic_n end_POSTSUBSCRIPT start_POSTSUPERSCRIPT * end_POSTSUPERSCRIPT italic_ϕ start_POSTSUBSCRIPT italic_n end_POSTSUBSCRIPT , italic_η start_POSTSUBSCRIPT italic_n end_POSTSUBSCRIPT ⟩ = ⟨ italic_ψ , italic_η ⟩

proving that ϕD(A*)italic-ϕ𝐷superscript𝐴\phi\in D(A^{*})italic_ϕ ∈ italic_D ( italic_A start_POSTSUPERSCRIPT * end_POSTSUPERSCRIPT ) and A*ϕ=ψsuperscript𝐴italic-ϕ𝜓A^{*}\phi=\psiitalic_A start_POSTSUPERSCRIPT * end_POSTSUPERSCRIPT italic_ϕ = italic_ψ as needed.

(b): This is a simple application of (a) and the fact that T**=Tsuperscript𝑇absent𝑇T^{**}=Titalic_T start_POSTSUPERSCRIPT * * end_POSTSUPERSCRIPT = italic_T for any closed operator T𝑇Titalic_T.

(c): We need only to prove that Gr(A)ΓGr𝐴subscriptΓ\operatorname{Gr}(A)\subseteq\Gamma_{\infty}roman_Gr ( italic_A ) ⊆ roman_Γ start_POSTSUBSCRIPT ∞ end_POSTSUBSCRIPT implies ΓGr(A)subscriptΓGr𝐴\Gamma_{\infty}\subseteq\operatorname{Gr}(A)roman_Γ start_POSTSUBSCRIPT ∞ end_POSTSUBSCRIPT ⊆ roman_Gr ( italic_A ). This is seen by the inclusions ΓΓ*subscriptΓsuperscriptsubscriptΓ\Gamma_{\infty}\subseteq\Gamma_{\infty}^{*}roman_Γ start_POSTSUBSCRIPT ∞ end_POSTSUBSCRIPT ⊆ roman_Γ start_POSTSUBSCRIPT ∞ end_POSTSUBSCRIPT start_POSTSUPERSCRIPT * end_POSTSUPERSCRIPT (by symmetry of the Ansubscript𝐴𝑛A_{n}italic_A start_POSTSUBSCRIPT italic_n end_POSTSUBSCRIPT’s) and Γ*Gr(A*)=Gr(A)superscriptsubscriptΓGrsuperscript𝐴Gr𝐴\Gamma_{\infty}^{*}\subseteq\operatorname{Gr}(A^{*})=\operatorname{Gr}(A)roman_Γ start_POSTSUBSCRIPT ∞ end_POSTSUBSCRIPT start_POSTSUPERSCRIPT * end_POSTSUPERSCRIPT ⊆ roman_Gr ( italic_A start_POSTSUPERSCRIPT * end_POSTSUPERSCRIPT ) = roman_Gr ( italic_A ) (by (a) and self-adjointness of A𝐴Aitalic_A). \square

The connection to convergence of the projections onto the graphs of the Ansubscript𝐴𝑛A_{n}italic_A start_POSTSUBSCRIPT italic_n end_POSTSUBSCRIPT’s is now given in the below proposition. It tells us that the difference between strong graph convergence and strong convergence of the sequence of graph projections is measured by the absence of strong graph convergence of the sequence of adjoint operators. {pro} Let {An}n=1superscriptsubscriptsubscript𝐴𝑛𝑛1\{A_{n}\}_{n=1}^{\infty}{ italic_A start_POSTSUBSCRIPT italic_n end_POSTSUBSCRIPT } start_POSTSUBSCRIPT italic_n = 1 end_POSTSUBSCRIPT start_POSTSUPERSCRIPT ∞ end_POSTSUPERSCRIPT be a sequence of densely defined and closed operators and let A𝐴Aitalic_A be an operator with the same properties as the Ansubscript𝐴𝑛A_{n}italic_A start_POSTSUBSCRIPT italic_n end_POSTSUBSCRIPT’s. Denote by Pnsubscript𝑃𝑛P_{n}italic_P start_POSTSUBSCRIPT italic_n end_POSTSUBSCRIPT and P𝑃Pitalic_P the orthogonal projections in direct-sum\mathcal{H}\oplus\mathcal{H}caligraphic_H ⊕ caligraphic_H onto Gr(An)Grsubscript𝐴𝑛\operatorname{Gr}(A_{n})roman_Gr ( italic_A start_POSTSUBSCRIPT italic_n end_POSTSUBSCRIPT ) and Gr(A)Gr𝐴\operatorname{Gr}(A)roman_Gr ( italic_A ) respectively. Then PnPsubscript𝑃𝑛𝑃P_{n}\to Pitalic_P start_POSTSUBSCRIPT italic_n end_POSTSUBSCRIPT → italic_P strongly if and only if both Gr(A)=ΓGr𝐴subscriptΓ\operatorname{Gr}(A)=\Gamma_{\infty}roman_Gr ( italic_A ) = roman_Γ start_POSTSUBSCRIPT ∞ end_POSTSUBSCRIPT and Gr(A*)=Γ*Grsuperscript𝐴superscriptsubscriptΓ\operatorname{Gr}(A^{*})=\Gamma_{\infty}^{*}roman_Gr ( italic_A start_POSTSUPERSCRIPT * end_POSTSUPERSCRIPT ) = roman_Γ start_POSTSUBSCRIPT ∞ end_POSTSUBSCRIPT start_POSTSUPERSCRIPT * end_POSTSUPERSCRIPT (or equivalently if and only if both Gr(A)ΓGr𝐴subscriptΓ\operatorname{Gr}(A)\subseteq\Gamma_{\infty}roman_Gr ( italic_A ) ⊆ roman_Γ start_POSTSUBSCRIPT ∞ end_POSTSUBSCRIPT and Gr(A*)Γ*Grsuperscript𝐴superscriptsubscriptΓ\operatorname{Gr}(A^{*})\subseteq\Gamma_{\infty}^{*}roman_Gr ( italic_A start_POSTSUPERSCRIPT * end_POSTSUPERSCRIPT ) ⊆ roman_Γ start_POSTSUBSCRIPT ∞ end_POSTSUBSCRIPT start_POSTSUPERSCRIPT * end_POSTSUPERSCRIPT, cf. Lemma Lemma(b) ).

Proof.

We will use the standard fact, see for example [4] Theorem 12.5, that

=Gr(T)WGr(T*)direct-sumdirect-sumGr𝑇𝑊Grsuperscript𝑇\mathcal{H}\oplus\mathcal{H}=\operatorname{Gr}(T)\oplus W\operatorname{Gr}(T^{% *})caligraphic_H ⊕ caligraphic_H = roman_Gr ( italic_T ) ⊕ italic_W roman_Gr ( italic_T start_POSTSUPERSCRIPT * end_POSTSUPERSCRIPT ) (2)

for any densely defined and closed operator T𝑇Titalic_T on \mathcal{H}caligraphic_H where the sum is orthogonal and W𝑊Witalic_W is the unitary map (ϕ,ψ)(ψ,ϕ)maps-toitalic-ϕ𝜓𝜓italic-ϕ(\phi,\psi)\mapsto(-\psi,\phi)( italic_ϕ , italic_ψ ) ↦ ( - italic_ψ , italic_ϕ ).

Now if PnPsubscript𝑃𝑛𝑃P_{n}\to Pitalic_P start_POSTSUBSCRIPT italic_n end_POSTSUBSCRIPT → italic_P strongly then Gr(A)=ΓGr𝐴subscriptΓ\operatorname{Gr}(A)=\Gamma_{\infty}roman_Gr ( italic_A ) = roman_Γ start_POSTSUBSCRIPT ∞ end_POSTSUBSCRIPT by Lemma Lemma(b). Also, 1Pn1P1subscript𝑃𝑛1𝑃1-P_{n}\to 1-P1 - italic_P start_POSTSUBSCRIPT italic_n end_POSTSUBSCRIPT → 1 - italic_P strongly so that similarly WGr(An*)WGr(A*)𝑊Grsuperscriptsubscript𝐴𝑛𝑊Grsuperscript𝐴W\operatorname{Gr}(A_{n}^{*})\to W\operatorname{Gr}(A^{*})italic_W roman_Gr ( italic_A start_POSTSUBSCRIPT italic_n end_POSTSUBSCRIPT start_POSTSUPERSCRIPT * end_POSTSUPERSCRIPT ) → italic_W roman_Gr ( italic_A start_POSTSUPERSCRIPT * end_POSTSUPERSCRIPT ) strongly by the decomposition (2). It is an easy exercise to check that this is equivalent to Gr(A*)=Γ*Grsuperscript𝐴superscriptsubscriptΓ\operatorname{Gr}(A^{*})=\Gamma_{\infty}^{*}roman_Gr ( italic_A start_POSTSUPERSCRIPT * end_POSTSUPERSCRIPT ) = roman_Γ start_POSTSUBSCRIPT ∞ end_POSTSUBSCRIPT start_POSTSUPERSCRIPT * end_POSTSUPERSCRIPT.

If, on the other hand, Gr(A)=ΓGr𝐴subscriptΓ\operatorname{Gr}(A)=\Gamma_{\infty}roman_Gr ( italic_A ) = roman_Γ start_POSTSUBSCRIPT ∞ end_POSTSUBSCRIPT and Gr(A*)=Γ*Grsuperscript𝐴superscriptsubscriptΓ\operatorname{Gr}(A^{*})=\Gamma_{\infty}^{*}roman_Gr ( italic_A start_POSTSUPERSCRIPT * end_POSTSUPERSCRIPT ) = roman_Γ start_POSTSUBSCRIPT ∞ end_POSTSUBSCRIPT start_POSTSUPERSCRIPT * end_POSTSUPERSCRIPT then also WGr(An*)WGr(A*)𝑊Grsuperscriptsubscript𝐴𝑛𝑊Grsuperscript𝐴W\operatorname{Gr}(A_{n}^{*})\to W\operatorname{Gr}(A^{*})italic_W roman_Gr ( italic_A start_POSTSUBSCRIPT italic_n end_POSTSUBSCRIPT start_POSTSUPERSCRIPT * end_POSTSUPERSCRIPT ) → italic_W roman_Gr ( italic_A start_POSTSUPERSCRIPT * end_POSTSUPERSCRIPT ) strongly. Using this we get by Lemma Lemma(a) that PnxPxsubscript𝑃𝑛𝑥𝑃𝑥P_{n}x\to Pxitalic_P start_POSTSUBSCRIPT italic_n end_POSTSUBSCRIPT italic_x → italic_P italic_x for any xGr(A)𝑥Gr𝐴x\in\operatorname{Gr}(A)italic_x ∈ roman_Gr ( italic_A ) and, by using additionally (2), (1Pn)y(1P)y1subscript𝑃𝑛𝑦1𝑃𝑦(1-P_{n})y\to(1-P)y( 1 - italic_P start_POSTSUBSCRIPT italic_n end_POSTSUBSCRIPT ) italic_y → ( 1 - italic_P ) italic_y for any yWGr(A*)𝑦𝑊Grsuperscript𝐴y\in W\operatorname{Gr}(A^{*})italic_y ∈ italic_W roman_Gr ( italic_A start_POSTSUPERSCRIPT * end_POSTSUPERSCRIPT ). Combining these convergences and (2) we conclude that PnzPzsubscript𝑃𝑛𝑧𝑃𝑧P_{n}z\to Pzitalic_P start_POSTSUBSCRIPT italic_n end_POSTSUBSCRIPT italic_z → italic_P italic_z for any z𝑧direct-sumz\in\mathcal{H}\oplus\mathcal{H}italic_z ∈ caligraphic_H ⊕ caligraphic_H. \square

To conclude this technical section we include a result on strong resolvent convergence of self-adjoint operators together with some observations. This result is well established, cf. [5] Lemma 28 (and [2] Theorem VIII.26 for a partial result). In the formulation below [5] states and proves the equivalence of (i), (ii) and (iv) and [2] that of (i) and (ii). Meanwhile, both proofs are sufficient to include also (iii) to these lists.

Theorem.

Let {Bn}n=1superscriptsubscriptsubscript𝐵𝑛𝑛1\{B_{n}\}_{n=1}^{\infty}{ italic_B start_POSTSUBSCRIPT italic_n end_POSTSUBSCRIPT } start_POSTSUBSCRIPT italic_n = 1 end_POSTSUBSCRIPT start_POSTSUPERSCRIPT ∞ end_POSTSUPERSCRIPT be a sequence of self-adjoint operators on a Hilbert space \mathcal{H}caligraphic_H and B𝐵Bitalic_B another self-adjoint operator on \mathcal{H}caligraphic_H. Let further Qnsubscript𝑄𝑛Q_{n}italic_Q start_POSTSUBSCRIPT italic_n end_POSTSUBSCRIPT and Q𝑄Qitalic_Q be the orthogonal projections onto Gr(Bn)Grsubscript𝐵𝑛\operatorname{Gr}(B_{n})roman_Gr ( italic_B start_POSTSUBSCRIPT italic_n end_POSTSUBSCRIPT ) and Gr(B)Gr𝐵\operatorname{Gr}(B)roman_Gr ( italic_B ) respectively and denote by ΓBsuperscriptsubscriptΓ𝐵\Gamma_{\infty}^{B}roman_Γ start_POSTSUBSCRIPT ∞ end_POSTSUBSCRIPT start_POSTSUPERSCRIPT italic_B end_POSTSUPERSCRIPT the strong limit of {Gr(Bn)}n=1superscriptsubscriptGrsubscript𝐵𝑛𝑛1\{\operatorname{Gr}(B_{n})\}_{n=1}^{\infty}{ roman_Gr ( italic_B start_POSTSUBSCRIPT italic_n end_POSTSUBSCRIPT ) } start_POSTSUBSCRIPT italic_n = 1 end_POSTSUBSCRIPT start_POSTSUPERSCRIPT ∞ end_POSTSUPERSCRIPT. The following statements are equivalent:

  • (i)

    BnBsubscript𝐵𝑛𝐵B_{n}\to Bitalic_B start_POSTSUBSCRIPT italic_n end_POSTSUBSCRIPT → italic_B in the strong resolvent sense,

  • (ii)

    B=str.gr.limBn𝐵formulae-sequencestrgrlimsubscript𝐵𝑛B=\operatorname{str.gr.lim}B_{n}italic_B = start_OPFUNCTION roman_str . roman_gr . roman_lim end_OPFUNCTION italic_B start_POSTSUBSCRIPT italic_n end_POSTSUBSCRIPT (i.e. Gr(B)=ΓBGr𝐵superscriptsubscriptΓ𝐵\operatorname{Gr}(B)=\Gamma_{\infty}^{B}roman_Gr ( italic_B ) = roman_Γ start_POSTSUBSCRIPT ∞ end_POSTSUBSCRIPT start_POSTSUPERSCRIPT italic_B end_POSTSUPERSCRIPT),

  • (iii)

    Gr(B)ΓBGr𝐵superscriptsubscriptΓ𝐵\operatorname{Gr}(B)\subseteq\Gamma_{\infty}^{B}roman_Gr ( italic_B ) ⊆ roman_Γ start_POSTSUBSCRIPT ∞ end_POSTSUBSCRIPT start_POSTSUPERSCRIPT italic_B end_POSTSUPERSCRIPT,

  • (iv)

    QnQsubscript𝑄𝑛𝑄Q_{n}\to Qitalic_Q start_POSTSUBSCRIPT italic_n end_POSTSUBSCRIPT → italic_Q strongly.

\square

Proof.

When using the self-adjointness of the operators, the equivalence between (ii) and (iii) is Lemma Lemma(c) and the equivalence between (ii) and (iv) is Proposition 2. Suppose BnBsubscript𝐵𝑛𝐵B_{n}\to Bitalic_B start_POSTSUBSCRIPT italic_n end_POSTSUBSCRIPT → italic_B in the strong resolvent sense and let ϕD(B)italic-ϕ𝐷𝐵\phi\in D(B)italic_ϕ ∈ italic_D ( italic_B ) be arbitrary. By self-adjointness the relation ψ=(Bn+i)ϕn=(B+i)ϕ𝜓subscript𝐵𝑛𝑖subscriptitalic-ϕ𝑛𝐵𝑖italic-ϕ\psi=(B_{n}+i)\phi_{n}=(B+i)\phiitalic_ψ = ( italic_B start_POSTSUBSCRIPT italic_n end_POSTSUBSCRIPT + italic_i ) italic_ϕ start_POSTSUBSCRIPT italic_n end_POSTSUBSCRIPT = ( italic_B + italic_i ) italic_ϕ defines besides the ψ𝜓\psi\in\mathcal{H}italic_ψ ∈ caligraphic_H also for each n𝑛nitalic_n a ϕnD(Bn)subscriptitalic-ϕ𝑛𝐷subscript𝐵𝑛\phi_{n}\in D(B_{n})italic_ϕ start_POSTSUBSCRIPT italic_n end_POSTSUBSCRIPT ∈ italic_D ( italic_B start_POSTSUBSCRIPT italic_n end_POSTSUBSCRIPT ). Moreover,

(ϕn,Bnϕn)(ϕ,Bϕ)2superscriptdelimited-∥∥subscriptitalic-ϕ𝑛subscript𝐵𝑛subscriptitalic-ϕ𝑛italic-ϕ𝐵italic-ϕ2\displaystyle\lVert(\phi_{n},B_{n}\phi_{n})-(\phi,B\phi)\rVert^{2}∥ ( italic_ϕ start_POSTSUBSCRIPT italic_n end_POSTSUBSCRIPT , italic_B start_POSTSUBSCRIPT italic_n end_POSTSUBSCRIPT italic_ϕ start_POSTSUBSCRIPT italic_n end_POSTSUBSCRIPT ) - ( italic_ϕ , italic_B italic_ϕ ) ∥ start_POSTSUPERSCRIPT 2 end_POSTSUPERSCRIPT =ϕnϕ2+BnϕnBϕ2=ϕnϕ2+iϕiϕn2absentsuperscriptdelimited-∥∥subscriptitalic-ϕ𝑛italic-ϕ2superscriptdelimited-∥∥subscript𝐵𝑛subscriptitalic-ϕ𝑛𝐵italic-ϕ2superscriptdelimited-∥∥subscriptitalic-ϕ𝑛italic-ϕ2superscriptdelimited-∥∥𝑖italic-ϕ𝑖subscriptitalic-ϕ𝑛2\displaystyle=\lVert\phi_{n}-\phi\rVert^{2}+\lVert B_{n}\phi_{n}-B\phi\rVert^{% 2}=\lVert\phi_{n}-\phi\rVert^{2}+\lVert i\phi-i\phi_{n}\rVert^{2}= ∥ italic_ϕ start_POSTSUBSCRIPT italic_n end_POSTSUBSCRIPT - italic_ϕ ∥ start_POSTSUPERSCRIPT 2 end_POSTSUPERSCRIPT + ∥ italic_B start_POSTSUBSCRIPT italic_n end_POSTSUBSCRIPT italic_ϕ start_POSTSUBSCRIPT italic_n end_POSTSUBSCRIPT - italic_B italic_ϕ ∥ start_POSTSUPERSCRIPT 2 end_POSTSUPERSCRIPT = ∥ italic_ϕ start_POSTSUBSCRIPT italic_n end_POSTSUBSCRIPT - italic_ϕ ∥ start_POSTSUPERSCRIPT 2 end_POSTSUPERSCRIPT + ∥ italic_i italic_ϕ - italic_i italic_ϕ start_POSTSUBSCRIPT italic_n end_POSTSUBSCRIPT ∥ start_POSTSUPERSCRIPT 2 end_POSTSUPERSCRIPT
=2(Bn+i)1ψ(B+i)1ψ20absent2superscriptdelimited-∥∥superscriptsubscript𝐵𝑛𝑖1𝜓superscript𝐵𝑖1𝜓20\displaystyle=2\lVert(B_{n}+i)^{-1}\psi-(B+i)^{-1}\psi\rVert^{2}\longrightarrow 0= 2 ∥ ( italic_B start_POSTSUBSCRIPT italic_n end_POSTSUBSCRIPT + italic_i ) start_POSTSUPERSCRIPT - 1 end_POSTSUPERSCRIPT italic_ψ - ( italic_B + italic_i ) start_POSTSUPERSCRIPT - 1 end_POSTSUPERSCRIPT italic_ψ ∥ start_POSTSUPERSCRIPT 2 end_POSTSUPERSCRIPT ⟶ 0

which proves that (i) implies (iii). Suppose finally that Gr(B)ΓBGr𝐵superscriptsubscriptΓ𝐵\operatorname{Gr}(B)\subseteq\Gamma_{\infty}^{B}roman_Gr ( italic_B ) ⊆ roman_Γ start_POSTSUBSCRIPT ∞ end_POSTSUBSCRIPT start_POSTSUPERSCRIPT italic_B end_POSTSUPERSCRIPT and let ψ𝜓\psi\in\mathcal{H}italic_ψ ∈ caligraphic_H be arbitrary. Since =R(B+i)𝑅𝐵𝑖\mathcal{H}=R(B+i)caligraphic_H = italic_R ( italic_B + italic_i ) we have ψ=(B+i)ϕ𝜓𝐵𝑖italic-ϕ\psi=(B+i)\phiitalic_ψ = ( italic_B + italic_i ) italic_ϕ for some ϕD(B)italic-ϕ𝐷𝐵\phi\in D(B)italic_ϕ ∈ italic_D ( italic_B ) and by the assumption there exists a sequence {ϕn}n=1superscriptsubscriptsubscriptitalic-ϕ𝑛𝑛1\{\phi_{n}\}_{n=1}^{\infty}\subseteq\mathcal{H}{ italic_ϕ start_POSTSUBSCRIPT italic_n end_POSTSUBSCRIPT } start_POSTSUBSCRIPT italic_n = 1 end_POSTSUBSCRIPT start_POSTSUPERSCRIPT ∞ end_POSTSUPERSCRIPT ⊆ caligraphic_H so that (ϕn,Bnϕn)(ϕ,Bϕ)subscriptitalic-ϕ𝑛subscript𝐵𝑛subscriptitalic-ϕ𝑛italic-ϕ𝐵italic-ϕ(\phi_{n},B_{n}\phi_{n})\to(\phi,B\phi)( italic_ϕ start_POSTSUBSCRIPT italic_n end_POSTSUBSCRIPT , italic_B start_POSTSUBSCRIPT italic_n end_POSTSUBSCRIPT italic_ϕ start_POSTSUBSCRIPT italic_n end_POSTSUBSCRIPT ) → ( italic_ϕ , italic_B italic_ϕ ). Hence,

[(Bn+i)1(B+i)1]ψ=(Bn+i)1[(B+i)ϕ(Bn+i)ϕn]ϕ+ϕn0delimited-[]superscriptsubscript𝐵𝑛𝑖1superscript𝐵𝑖1𝜓superscriptsubscript𝐵𝑛𝑖1delimited-[]𝐵𝑖italic-ϕsubscript𝐵𝑛𝑖subscriptitalic-ϕ𝑛italic-ϕsubscriptitalic-ϕ𝑛0[(B_{n}+i)^{-1}-(B+i)^{-1}]\psi=(B_{n}+i)^{-1}[(B+i)\phi-(B_{n}+i)\phi_{n}]-% \phi+\phi_{n}\longrightarrow 0[ ( italic_B start_POSTSUBSCRIPT italic_n end_POSTSUBSCRIPT + italic_i ) start_POSTSUPERSCRIPT - 1 end_POSTSUPERSCRIPT - ( italic_B + italic_i ) start_POSTSUPERSCRIPT - 1 end_POSTSUPERSCRIPT ] italic_ψ = ( italic_B start_POSTSUBSCRIPT italic_n end_POSTSUBSCRIPT + italic_i ) start_POSTSUPERSCRIPT - 1 end_POSTSUPERSCRIPT [ ( italic_B + italic_i ) italic_ϕ - ( italic_B start_POSTSUBSCRIPT italic_n end_POSTSUBSCRIPT + italic_i ) italic_ϕ start_POSTSUBSCRIPT italic_n end_POSTSUBSCRIPT ] - italic_ϕ + italic_ϕ start_POSTSUBSCRIPT italic_n end_POSTSUBSCRIPT ⟶ 0

where we use the fact that (Bn+i)11delimited-∥∥superscriptsubscript𝐵𝑛𝑖11\lVert(B_{n}+i)^{-1}\rVert\leq 1∥ ( italic_B start_POSTSUBSCRIPT italic_n end_POSTSUBSCRIPT + italic_i ) start_POSTSUPERSCRIPT - 1 end_POSTSUPERSCRIPT ∥ ≤ 1 for all n𝑛nitalic_n. This proves that (iii) implies (i) and thus the full theorem. \square

We observe that though (ii)-(iv) in Theorem Theorem are equivalent for sequences of self-adjoint operators, Proposition 2 tells us that (iii) follows from (ii) and (iv) respectively even when assuming only that the Bnsubscript𝐵𝑛B_{n}italic_B start_POSTSUBSCRIPT italic_n end_POSTSUBSCRIPT’s and B𝐵Bitalic_B are densely defined and closed. Moreover, (iii) is a consequence of pointwise convergence on a common core of the sequence and is thus easy to verify for for example differential operators, see Proposition 3 and Example Example. Thus, the first question we examine in Section 3 below will be the following: If we in the set-up from Section 1 impose the condition (iii) on the sequence {An}n=1superscriptsubscriptsubscript𝐴𝑛𝑛1\{A_{n}\}_{n=1}^{\infty}{ italic_A start_POSTSUBSCRIPT italic_n end_POSTSUBSCRIPT } start_POSTSUBSCRIPT italic_n = 1 end_POSTSUBSCRIPT start_POSTSUPERSCRIPT ∞ end_POSTSUPERSCRIPT what more do we need in order for it to hold for the sequence {Bn}n=1superscriptsubscriptsubscript𝐵𝑛𝑛1\{B_{n}\}_{n=1}^{\infty}{ italic_B start_POSTSUBSCRIPT italic_n end_POSTSUBSCRIPT } start_POSTSUBSCRIPT italic_n = 1 end_POSTSUBSCRIPT start_POSTSUPERSCRIPT ∞ end_POSTSUPERSCRIPT of extensions (thus yielding strong resolvent convergence)?

3 Main results

From the previous section we obtain two candidates for convergence type to impose on the Ansubscript𝐴𝑛A_{n}italic_A start_POSTSUBSCRIPT italic_n end_POSTSUBSCRIPT’s in (1): Strong graph convergence and strong convergence of graph projections. That these are actually both natural choices is illuminated by Theorem Theorem which states that for sequences of self-adjoint operators each of them is equivalent to strong resolvent convergence – exactly the convergence type we seek! Throughout this section we use the following conventions: Let {An}n=1superscriptsubscriptsubscript𝐴𝑛𝑛1\{A_{n}\}_{n=1}^{\infty}{ italic_A start_POSTSUBSCRIPT italic_n end_POSTSUBSCRIPT } start_POSTSUBSCRIPT italic_n = 1 end_POSTSUBSCRIPT start_POSTSUPERSCRIPT ∞ end_POSTSUPERSCRIPT, {Bn}n=1superscriptsubscriptsubscript𝐵𝑛𝑛1\{B_{n}\}_{n=1}^{\infty}{ italic_B start_POSTSUBSCRIPT italic_n end_POSTSUBSCRIPT } start_POSTSUBSCRIPT italic_n = 1 end_POSTSUBSCRIPT start_POSTSUPERSCRIPT ∞ end_POSTSUPERSCRIPT, {Un}n=1superscriptsubscriptsubscript𝑈𝑛𝑛1\{U_{n}\}_{n=1}^{\infty}{ italic_U start_POSTSUBSCRIPT italic_n end_POSTSUBSCRIPT } start_POSTSUBSCRIPT italic_n = 1 end_POSTSUBSCRIPT start_POSTSUPERSCRIPT ∞ end_POSTSUPERSCRIPT, A𝐴Aitalic_A, B𝐵Bitalic_B and U𝑈Uitalic_U be as in the beginning of Section 1. Let ΓsubscriptΓ\Gamma_{\infty}roman_Γ start_POSTSUBSCRIPT ∞ end_POSTSUBSCRIPT be the strong limit of {Gr(An)}n=1superscriptsubscriptGrsubscript𝐴𝑛𝑛1\{\operatorname{Gr}(A_{n})\}_{n=1}^{\infty}{ roman_Gr ( italic_A start_POSTSUBSCRIPT italic_n end_POSTSUBSCRIPT ) } start_POSTSUBSCRIPT italic_n = 1 end_POSTSUBSCRIPT start_POSTSUPERSCRIPT ∞ end_POSTSUPERSCRIPT and Γ*superscriptsubscriptΓ\Gamma_{\infty}^{*}roman_Γ start_POSTSUBSCRIPT ∞ end_POSTSUBSCRIPT start_POSTSUPERSCRIPT * end_POSTSUPERSCRIPT the strong limit of {Gr(An*)}n=1superscriptsubscriptGrsuperscriptsubscript𝐴𝑛𝑛1\{\operatorname{Gr}(A_{n}^{*})\}_{n=1}^{\infty}{ roman_Gr ( italic_A start_POSTSUBSCRIPT italic_n end_POSTSUBSCRIPT start_POSTSUPERSCRIPT * end_POSTSUPERSCRIPT ) } start_POSTSUBSCRIPT italic_n = 1 end_POSTSUBSCRIPT start_POSTSUPERSCRIPT ∞ end_POSTSUPERSCRIPT. Denote by Pnsubscript𝑃𝑛P_{n}italic_P start_POSTSUBSCRIPT italic_n end_POSTSUBSCRIPT and P𝑃Pitalic_P the orthogonal projections in direct-sum\mathcal{H}\oplus\mathcal{H}caligraphic_H ⊕ caligraphic_H onto Gr(An)Grsubscript𝐴𝑛\operatorname{Gr}(A_{n})roman_Gr ( italic_A start_POSTSUBSCRIPT italic_n end_POSTSUBSCRIPT ) and Gr(A)Gr𝐴\operatorname{Gr}(A)roman_Gr ( italic_A ) respectively.

The answer to the question closing Section 2 is straightforward and given below in Corollary Corollary, and in applications it can be useful even in this raw form.

Corollary (to Theorem Theorem).

Consider operators Ansubscript𝐴𝑛A_{n}italic_A start_POSTSUBSCRIPT italic_n end_POSTSUBSCRIPT, A𝐴Aitalic_A, Bnsubscript𝐵𝑛B_{n}italic_B start_POSTSUBSCRIPT italic_n end_POSTSUBSCRIPT and B𝐵Bitalic_B as described in Section 1 and suppose Gr(A)ΓGr𝐴subscriptΓ\operatorname{Gr}(A)\subseteq\Gamma_{\infty}roman_Gr ( italic_A ) ⊆ roman_Γ start_POSTSUBSCRIPT ∞ end_POSTSUBSCRIPT. If moreover for every pair (or, equivalently, for k𝑘kitalic_k linearly independent pairs) (ϕ,Bϕ)italic-ϕ𝐵italic-ϕ(\phi,B\phi)( italic_ϕ , italic_B italic_ϕ ) from the orthogonal complement of Gr(A)Gr𝐴\operatorname{Gr}(A)roman_Gr ( italic_A ) inside the Hilbert space Gr(B)Gr𝐵\operatorname{Gr}(B)roman_Gr ( italic_B ) there exists a sequence {ϕn}n=1superscriptsubscriptsubscriptitalic-ϕ𝑛𝑛1\{\phi_{n}\}_{n=1}^{\infty}\subseteq\mathcal{H}{ italic_ϕ start_POSTSUBSCRIPT italic_n end_POSTSUBSCRIPT } start_POSTSUBSCRIPT italic_n = 1 end_POSTSUBSCRIPT start_POSTSUPERSCRIPT ∞ end_POSTSUPERSCRIPT ⊆ caligraphic_H so that ϕnD(Bn)subscriptitalic-ϕ𝑛𝐷subscript𝐵𝑛\phi_{n}\in D(B_{n})italic_ϕ start_POSTSUBSCRIPT italic_n end_POSTSUBSCRIPT ∈ italic_D ( italic_B start_POSTSUBSCRIPT italic_n end_POSTSUBSCRIPT ) for all n𝑛nitalic_n and (ϕn,Bnϕn)(ϕ,Bϕ)subscriptitalic-ϕ𝑛subscript𝐵𝑛subscriptitalic-ϕ𝑛italic-ϕ𝐵italic-ϕ(\phi_{n},B_{n}\phi_{n})\to(\phi,B\phi)( italic_ϕ start_POSTSUBSCRIPT italic_n end_POSTSUBSCRIPT , italic_B start_POSTSUBSCRIPT italic_n end_POSTSUBSCRIPT italic_ϕ start_POSTSUBSCRIPT italic_n end_POSTSUBSCRIPT ) → ( italic_ϕ , italic_B italic_ϕ ) then BnBsubscript𝐵𝑛𝐵B_{n}\to Bitalic_B start_POSTSUBSCRIPT italic_n end_POSTSUBSCRIPT → italic_B in the strong resolvent sense. \square

Proof.

Denoting the strong limit of {Gr(Bn)}n=1superscriptsubscriptGrsubscript𝐵𝑛𝑛1\{\operatorname{Gr}(B_{n})\}_{n=1}^{\infty}{ roman_Gr ( italic_B start_POSTSUBSCRIPT italic_n end_POSTSUBSCRIPT ) } start_POSTSUBSCRIPT italic_n = 1 end_POSTSUBSCRIPT start_POSTSUPERSCRIPT ∞ end_POSTSUPERSCRIPT by ΓBsuperscriptsubscriptΓ𝐵\Gamma_{\infty}^{B}roman_Γ start_POSTSUBSCRIPT ∞ end_POSTSUBSCRIPT start_POSTSUPERSCRIPT italic_B end_POSTSUPERSCRIPT it is basically the assumption above that the orthogonal complement of Gr(A)Gr𝐴\operatorname{Gr}(A)roman_Gr ( italic_A ) inside Gr(B)Gr𝐵\operatorname{Gr}(B)roman_Gr ( italic_B ) is contained in ΓBsuperscriptsubscriptΓ𝐵\Gamma_{\infty}^{B}roman_Γ start_POSTSUBSCRIPT ∞ end_POSTSUBSCRIPT start_POSTSUPERSCRIPT italic_B end_POSTSUPERSCRIPT. Moreover, Gr(A)ΓΓBGr𝐴subscriptΓsuperscriptsubscriptΓ𝐵\operatorname{Gr}(A)\subseteq\Gamma_{\infty}\subseteq\Gamma_{\infty}^{B}roman_Gr ( italic_A ) ⊆ roman_Γ start_POSTSUBSCRIPT ∞ end_POSTSUBSCRIPT ⊆ roman_Γ start_POSTSUBSCRIPT ∞ end_POSTSUBSCRIPT start_POSTSUPERSCRIPT italic_B end_POSTSUPERSCRIPT since all the Bnsubscript𝐵𝑛B_{n}italic_B start_POSTSUBSCRIPT italic_n end_POSTSUBSCRIPT’s are extensions of the Ansubscript𝐴𝑛A_{n}italic_A start_POSTSUBSCRIPT italic_n end_POSTSUBSCRIPT’s. This concludes the proof. For the fact that it suffices to consider k𝑘kitalic_k linearly independent pairs, see the first couple of lines of the proof of Theorem Theorem. \square

For the remaining part of this section we formulate and prove results like (1) with the different notions of convergence of the Ansubscript𝐴𝑛A_{n}italic_A start_POSTSUBSCRIPT italic_n end_POSTSUBSCRIPT’s introduced above. For this it will be essential to have at our disposal the following characterization of strong convergence of the Unsubscript𝑈𝑛U_{n}italic_U start_POSTSUBSCRIPT italic_n end_POSTSUBSCRIPT’s defining the self-adjoint extensions of the Ansubscript𝐴𝑛A_{n}italic_A start_POSTSUBSCRIPT italic_n end_POSTSUBSCRIPT’s.

Lemma.

Consider the Unsubscript𝑈𝑛U_{n}italic_U start_POSTSUBSCRIPT italic_n end_POSTSUBSCRIPT’s and the U𝑈Uitalic_U described in Section 1. We have UnUsubscript𝑈𝑛𝑈U_{n}\to Uitalic_U start_POSTSUBSCRIPT italic_n end_POSTSUBSCRIPT → italic_U strongly if and only if the following statement is true:

For each ϕ++there exists a sequence {ϕ+n}n=1 so thatϕ+n+n for all n and (ϕ+n,Unϕ+n)(ϕ+,Uϕ+).For each ϕ++there exists a sequence {ϕ+n}n=1 so thatϕ+n+n for all n and (ϕ+n,Unϕ+n)(ϕ+,Uϕ+).\begin{gathered}\text{For each $\phi_{+}\in\mathcal{H}_{+}$there exists a % sequence $\{\phi_{+}^{n}\}_{n=1}^{\infty}\subseteq\mathcal{H}$ so that}\\ \text{$\phi_{+}^{n}\in\mathcal{H}_{+}^{n}$ for all $n$ and $(\phi_{+}^{n},U_{n% }\phi_{+}^{n})\to(\phi_{+},U\phi_{+})$.}\end{gathered}start_ROW start_CELL For each italic_ϕ start_POSTSUBSCRIPT + end_POSTSUBSCRIPT ∈ caligraphic_H start_POSTSUBSCRIPT + end_POSTSUBSCRIPT there exists a sequence { italic_ϕ start_POSTSUBSCRIPT + end_POSTSUBSCRIPT start_POSTSUPERSCRIPT italic_n end_POSTSUPERSCRIPT } start_POSTSUBSCRIPT italic_n = 1 end_POSTSUBSCRIPT start_POSTSUPERSCRIPT ∞ end_POSTSUPERSCRIPT ⊆ caligraphic_H so that end_CELL end_ROW start_ROW start_CELL italic_ϕ start_POSTSUBSCRIPT + end_POSTSUBSCRIPT start_POSTSUPERSCRIPT italic_n end_POSTSUPERSCRIPT ∈ caligraphic_H start_POSTSUBSCRIPT + end_POSTSUBSCRIPT start_POSTSUPERSCRIPT italic_n end_POSTSUPERSCRIPT for all italic_n and ( italic_ϕ start_POSTSUBSCRIPT + end_POSTSUBSCRIPT start_POSTSUPERSCRIPT italic_n end_POSTSUPERSCRIPT , italic_U start_POSTSUBSCRIPT italic_n end_POSTSUBSCRIPT italic_ϕ start_POSTSUBSCRIPT + end_POSTSUBSCRIPT start_POSTSUPERSCRIPT italic_n end_POSTSUPERSCRIPT ) → ( italic_ϕ start_POSTSUBSCRIPT + end_POSTSUBSCRIPT , italic_U italic_ϕ start_POSTSUBSCRIPT + end_POSTSUBSCRIPT ) . end_CELL end_ROW (3)

\square

Note that the condition (3) actually says that the strong limit of the graphs of the Unsubscript𝑈𝑛U_{n}italic_U start_POSTSUBSCRIPT italic_n end_POSTSUBSCRIPT’s considered as operators only on +nsuperscriptsubscript𝑛\mathcal{H}_{+}^{n}caligraphic_H start_POSTSUBSCRIPT + end_POSTSUBSCRIPT start_POSTSUPERSCRIPT italic_n end_POSTSUPERSCRIPT contains the corresponding graph of U𝑈Uitalic_U.

Proof (of Lemma Lemma).

Observe firstly that if ψnψsubscript𝜓𝑛𝜓\psi_{n}\to\psiitalic_ψ start_POSTSUBSCRIPT italic_n end_POSTSUBSCRIPT → italic_ψ then the inequalities

UnψnUψψnψ+UnψUψ2ψnψ+UnψnUψdelimited-∥∥subscript𝑈𝑛subscript𝜓𝑛𝑈𝜓delimited-∥∥subscript𝜓𝑛𝜓delimited-∥∥subscript𝑈𝑛𝜓𝑈𝜓2delimited-∥∥subscript𝜓𝑛𝜓delimited-∥∥subscript𝑈𝑛subscript𝜓𝑛𝑈𝜓\lVert U_{n}\psi_{n}-U\psi\rVert\leq\lVert\psi_{n}-\psi\rVert+\lVert U_{n}\psi% -U\psi\rVert\leq 2\lVert\psi_{n}-\psi\rVert+\lVert U_{n}\psi_{n}-U\psi\rVert∥ italic_U start_POSTSUBSCRIPT italic_n end_POSTSUBSCRIPT italic_ψ start_POSTSUBSCRIPT italic_n end_POSTSUBSCRIPT - italic_U italic_ψ ∥ ≤ ∥ italic_ψ start_POSTSUBSCRIPT italic_n end_POSTSUBSCRIPT - italic_ψ ∥ + ∥ italic_U start_POSTSUBSCRIPT italic_n end_POSTSUBSCRIPT italic_ψ - italic_U italic_ψ ∥ ≤ 2 ∥ italic_ψ start_POSTSUBSCRIPT italic_n end_POSTSUBSCRIPT - italic_ψ ∥ + ∥ italic_U start_POSTSUBSCRIPT italic_n end_POSTSUBSCRIPT italic_ψ start_POSTSUBSCRIPT italic_n end_POSTSUBSCRIPT - italic_U italic_ψ ∥

show that

UnψnUψsubscript𝑈𝑛subscript𝜓𝑛𝑈𝜓U_{n}\psi_{n}\to U\psiitalic_U start_POSTSUBSCRIPT italic_n end_POSTSUBSCRIPT italic_ψ start_POSTSUBSCRIPT italic_n end_POSTSUBSCRIPT → italic_U italic_ψ    if and only if    UnψUψsubscript𝑈𝑛𝜓𝑈𝜓U_{n}\psi\to U\psiitalic_U start_POSTSUBSCRIPT italic_n end_POSTSUBSCRIPT italic_ψ → italic_U italic_ψ. (4)

For each n𝑛nitalic_n denote by Pnsubscript𝑃𝑛P_{n}italic_P start_POSTSUBSCRIPT italic_n end_POSTSUBSCRIPT the orthogonal projection onto +nsuperscriptsubscript𝑛\mathcal{H}_{+}^{n}caligraphic_H start_POSTSUBSCRIPT + end_POSTSUBSCRIPT start_POSTSUPERSCRIPT italic_n end_POSTSUPERSCRIPT and by P𝑃Pitalic_P the orthogonal projection onto +subscript\mathcal{H}_{+}caligraphic_H start_POSTSUBSCRIPT + end_POSTSUBSCRIPT. Assume UnUsubscript𝑈𝑛𝑈U_{n}\to Uitalic_U start_POSTSUBSCRIPT italic_n end_POSTSUBSCRIPT → italic_U strongly. Then, for any ϕ++subscriptitalic-ϕsubscript\phi_{+}\in\mathcal{H}_{+}italic_ϕ start_POSTSUBSCRIPT + end_POSTSUBSCRIPT ∈ caligraphic_H start_POSTSUBSCRIPT + end_POSTSUBSCRIPT and ψ𝜓\psi\in\mathcal{H}italic_ψ ∈ caligraphic_H, we have

Pnϕ+,ψ=Un*Unϕ+,ψ=Unϕ+,UnψUϕ+,Uψ=Pϕ+,ψ=ϕ+,ψsubscript𝑃𝑛subscriptitalic-ϕ𝜓superscriptsubscript𝑈𝑛subscript𝑈𝑛subscriptitalic-ϕ𝜓subscript𝑈𝑛subscriptitalic-ϕsubscript𝑈𝑛𝜓𝑈subscriptitalic-ϕ𝑈𝜓𝑃subscriptitalic-ϕ𝜓subscriptitalic-ϕ𝜓\langle P_{n}\phi_{+},\psi\rangle=\langle U_{n}^{*}U_{n}\phi_{+},\psi\rangle=% \langle U_{n}\phi_{+},U_{n}\psi\rangle\longrightarrow\langle U\phi_{+},U\psi% \rangle=\langle P\phi_{+},\psi\rangle=\langle\phi_{+},\psi\rangle⟨ italic_P start_POSTSUBSCRIPT italic_n end_POSTSUBSCRIPT italic_ϕ start_POSTSUBSCRIPT + end_POSTSUBSCRIPT , italic_ψ ⟩ = ⟨ italic_U start_POSTSUBSCRIPT italic_n end_POSTSUBSCRIPT start_POSTSUPERSCRIPT * end_POSTSUPERSCRIPT italic_U start_POSTSUBSCRIPT italic_n end_POSTSUBSCRIPT italic_ϕ start_POSTSUBSCRIPT + end_POSTSUBSCRIPT , italic_ψ ⟩ = ⟨ italic_U start_POSTSUBSCRIPT italic_n end_POSTSUBSCRIPT italic_ϕ start_POSTSUBSCRIPT + end_POSTSUBSCRIPT , italic_U start_POSTSUBSCRIPT italic_n end_POSTSUBSCRIPT italic_ψ ⟩ ⟶ ⟨ italic_U italic_ϕ start_POSTSUBSCRIPT + end_POSTSUBSCRIPT , italic_U italic_ψ ⟩ = ⟨ italic_P italic_ϕ start_POSTSUBSCRIPT + end_POSTSUBSCRIPT , italic_ψ ⟩ = ⟨ italic_ϕ start_POSTSUBSCRIPT + end_POSTSUBSCRIPT , italic_ψ ⟩

so that Pnϕ+ϕ+subscript𝑃𝑛subscriptitalic-ϕsubscriptitalic-ϕP_{n}\phi_{+}\to\phi_{+}italic_P start_POSTSUBSCRIPT italic_n end_POSTSUBSCRIPT italic_ϕ start_POSTSUBSCRIPT + end_POSTSUBSCRIPT → italic_ϕ start_POSTSUBSCRIPT + end_POSTSUBSCRIPT weakly in \mathcal{H}caligraphic_H. As further

ϕ+lim infnPnϕ+lim supnPnϕ+ϕ+delimited-∥∥subscriptitalic-ϕsubscriptlimit-infimum𝑛delimited-∥∥subscript𝑃𝑛subscriptitalic-ϕsubscriptlimit-supremum𝑛delimited-∥∥subscript𝑃𝑛subscriptitalic-ϕdelimited-∥∥subscriptitalic-ϕ\lVert\phi_{+}\rVert\leq\liminf_{n\to\infty}\lVert P_{n}\phi_{+}\rVert\leq% \limsup_{n\to\infty}\lVert P_{n}\phi_{+}\rVert\leq\lVert\phi_{+}\rVert∥ italic_ϕ start_POSTSUBSCRIPT + end_POSTSUBSCRIPT ∥ ≤ lim inf start_POSTSUBSCRIPT italic_n → ∞ end_POSTSUBSCRIPT ∥ italic_P start_POSTSUBSCRIPT italic_n end_POSTSUBSCRIPT italic_ϕ start_POSTSUBSCRIPT + end_POSTSUBSCRIPT ∥ ≤ lim sup start_POSTSUBSCRIPT italic_n → ∞ end_POSTSUBSCRIPT ∥ italic_P start_POSTSUBSCRIPT italic_n end_POSTSUBSCRIPT italic_ϕ start_POSTSUBSCRIPT + end_POSTSUBSCRIPT ∥ ≤ ∥ italic_ϕ start_POSTSUBSCRIPT + end_POSTSUBSCRIPT ∥

by lower semi-continuity of the norm it is apparent that additionally Pnϕ+ϕ+delimited-∥∥subscript𝑃𝑛subscriptitalic-ϕdelimited-∥∥subscriptitalic-ϕ\lVert P_{n}\phi_{+}\rVert\to\lVert\phi_{+}\rVert∥ italic_P start_POSTSUBSCRIPT italic_n end_POSTSUBSCRIPT italic_ϕ start_POSTSUBSCRIPT + end_POSTSUBSCRIPT ∥ → ∥ italic_ϕ start_POSTSUBSCRIPT + end_POSTSUBSCRIPT ∥ and consequently Pnϕ+ϕ+subscript𝑃𝑛subscriptitalic-ϕsubscriptitalic-ϕP_{n}\phi_{+}\to\phi_{+}italic_P start_POSTSUBSCRIPT italic_n end_POSTSUBSCRIPT italic_ϕ start_POSTSUBSCRIPT + end_POSTSUBSCRIPT → italic_ϕ start_POSTSUBSCRIPT + end_POSTSUBSCRIPT with respect to the Hilbert space norm. We claim that letting {ϕ+n}n=1:={Pnϕ+}n=1assignsuperscriptsubscriptsuperscriptsubscriptitalic-ϕ𝑛𝑛1superscriptsubscriptsubscript𝑃𝑛subscriptitalic-ϕ𝑛1\{\phi_{+}^{n}\}_{n=1}^{\infty}:=\{P_{n}\phi_{+}\}_{n=1}^{\infty}{ italic_ϕ start_POSTSUBSCRIPT + end_POSTSUBSCRIPT start_POSTSUPERSCRIPT italic_n end_POSTSUPERSCRIPT } start_POSTSUBSCRIPT italic_n = 1 end_POSTSUBSCRIPT start_POSTSUPERSCRIPT ∞ end_POSTSUPERSCRIPT := { italic_P start_POSTSUBSCRIPT italic_n end_POSTSUBSCRIPT italic_ϕ start_POSTSUBSCRIPT + end_POSTSUBSCRIPT } start_POSTSUBSCRIPT italic_n = 1 end_POSTSUBSCRIPT start_POSTSUPERSCRIPT ∞ end_POSTSUPERSCRIPT for each ϕ++subscriptitalic-ϕsubscript\phi_{+}\in\mathcal{H}_{+}italic_ϕ start_POSTSUBSCRIPT + end_POSTSUBSCRIPT ∈ caligraphic_H start_POSTSUBSCRIPT + end_POSTSUBSCRIPT verifies (3). Indeed, since then ϕ+nϕ+superscriptsubscriptitalic-ϕ𝑛subscriptitalic-ϕ\phi_{+}^{n}\to\phi_{+}italic_ϕ start_POSTSUBSCRIPT + end_POSTSUBSCRIPT start_POSTSUPERSCRIPT italic_n end_POSTSUPERSCRIPT → italic_ϕ start_POSTSUBSCRIPT + end_POSTSUBSCRIPT, the strong convergence UnUsubscript𝑈𝑛𝑈U_{n}\to Uitalic_U start_POSTSUBSCRIPT italic_n end_POSTSUBSCRIPT → italic_U and (4) yield the desired conclusion.

Suppose now that (3) is satisfied. For any ϕ++subscriptitalic-ϕsubscript\phi_{+}\in\mathcal{H}_{+}italic_ϕ start_POSTSUBSCRIPT + end_POSTSUBSCRIPT ∈ caligraphic_H start_POSTSUBSCRIPT + end_POSTSUBSCRIPT we can choose the sequence from this condition and (4) implies that Unϕ+Uϕ+subscript𝑈𝑛subscriptitalic-ϕ𝑈subscriptitalic-ϕU_{n}\phi_{+}\to U\phi_{+}italic_U start_POSTSUBSCRIPT italic_n end_POSTSUBSCRIPT italic_ϕ start_POSTSUBSCRIPT + end_POSTSUBSCRIPT → italic_U italic_ϕ start_POSTSUBSCRIPT + end_POSTSUBSCRIPT. For proving convergence of Unψsubscript𝑈𝑛𝜓U_{n}\psiitalic_U start_POSTSUBSCRIPT italic_n end_POSTSUBSCRIPT italic_ψ for ψ[+]𝜓superscriptdelimited-[]subscriptperpendicular-to\psi\in[\mathcal{H}_{+}]^{\perp}italic_ψ ∈ [ caligraphic_H start_POSTSUBSCRIPT + end_POSTSUBSCRIPT ] start_POSTSUPERSCRIPT ⟂ end_POSTSUPERSCRIPT fix such ψ𝜓\psiitalic_ψ and consider an orthonormal basis {ϕ+,1,,ϕ+,k}subscriptitalic-ϕ1subscriptitalic-ϕ𝑘\{\phi_{+,1},\dots,\phi_{+,k}\}{ italic_ϕ start_POSTSUBSCRIPT + , 1 end_POSTSUBSCRIPT , … , italic_ϕ start_POSTSUBSCRIPT + , italic_k end_POSTSUBSCRIPT } of +subscript\mathcal{H}_{+}caligraphic_H start_POSTSUBSCRIPT + end_POSTSUBSCRIPT. By (3) there exist sequences {ϕ+,1n}n=1,,{ϕ+,kn}n=1superscriptsubscriptsuperscriptsubscriptitalic-ϕ1𝑛𝑛1superscriptsubscriptsuperscriptsubscriptitalic-ϕ𝑘𝑛𝑛1\{\phi_{+,1}^{n}\}_{n=1}^{\infty},\dots,\{\phi_{+,k}^{n}\}_{n=1}^{\infty}% \subseteq\mathcal{H}{ italic_ϕ start_POSTSUBSCRIPT + , 1 end_POSTSUBSCRIPT start_POSTSUPERSCRIPT italic_n end_POSTSUPERSCRIPT } start_POSTSUBSCRIPT italic_n = 1 end_POSTSUBSCRIPT start_POSTSUPERSCRIPT ∞ end_POSTSUPERSCRIPT , … , { italic_ϕ start_POSTSUBSCRIPT + , italic_k end_POSTSUBSCRIPT start_POSTSUPERSCRIPT italic_n end_POSTSUPERSCRIPT } start_POSTSUBSCRIPT italic_n = 1 end_POSTSUBSCRIPT start_POSTSUPERSCRIPT ∞ end_POSTSUPERSCRIPT ⊆ caligraphic_H with ϕ+,n+nsuperscriptsubscriptitalic-ϕ𝑛superscriptsubscript𝑛\phi_{+,\ell}^{n}\in\mathcal{H}_{+}^{n}italic_ϕ start_POSTSUBSCRIPT + , roman_ℓ end_POSTSUBSCRIPT start_POSTSUPERSCRIPT italic_n end_POSTSUPERSCRIPT ∈ caligraphic_H start_POSTSUBSCRIPT + end_POSTSUBSCRIPT start_POSTSUPERSCRIPT italic_n end_POSTSUPERSCRIPT for all n𝑛nitalic_n and =1,,k1𝑘\ell=1,\dots,kroman_ℓ = 1 , … , italic_k and ϕ+,nϕ+,superscriptsubscriptitalic-ϕ𝑛subscriptitalic-ϕ\phi_{+,\ell}^{n}\to\phi_{+,\ell}italic_ϕ start_POSTSUBSCRIPT + , roman_ℓ end_POSTSUBSCRIPT start_POSTSUPERSCRIPT italic_n end_POSTSUPERSCRIPT → italic_ϕ start_POSTSUBSCRIPT + , roman_ℓ end_POSTSUBSCRIPT for all \ellroman_ℓ. Now by applying the Gram-Schmidt process to {ϕ+,1n,,ϕ+,kn}superscriptsubscriptitalic-ϕ1𝑛superscriptsubscriptitalic-ϕ𝑘𝑛\{\phi_{+,1}^{n},\dots,\phi_{+,k}^{n}\}{ italic_ϕ start_POSTSUBSCRIPT + , 1 end_POSTSUBSCRIPT start_POSTSUPERSCRIPT italic_n end_POSTSUPERSCRIPT , … , italic_ϕ start_POSTSUBSCRIPT + , italic_k end_POSTSUBSCRIPT start_POSTSUPERSCRIPT italic_n end_POSTSUPERSCRIPT } for each n𝑛nitalic_n we obtain new sequences {ϕ~+,1n}n=1,,{ϕ~+,kn}n=1superscriptsubscriptsuperscriptsubscript~italic-ϕ1𝑛𝑛1superscriptsubscriptsuperscriptsubscript~italic-ϕ𝑘𝑛𝑛1\{\widetilde{\phi}_{+,1}^{n}\}_{n=1}^{\infty},\dots,\{\widetilde{\phi}_{+,k}^{% n}\}_{n=1}^{\infty}\subseteq\mathcal{H}{ over~ start_ARG italic_ϕ end_ARG start_POSTSUBSCRIPT + , 1 end_POSTSUBSCRIPT start_POSTSUPERSCRIPT italic_n end_POSTSUPERSCRIPT } start_POSTSUBSCRIPT italic_n = 1 end_POSTSUBSCRIPT start_POSTSUPERSCRIPT ∞ end_POSTSUPERSCRIPT , … , { over~ start_ARG italic_ϕ end_ARG start_POSTSUBSCRIPT + , italic_k end_POSTSUBSCRIPT start_POSTSUPERSCRIPT italic_n end_POSTSUPERSCRIPT } start_POSTSUBSCRIPT italic_n = 1 end_POSTSUBSCRIPT start_POSTSUPERSCRIPT ∞ end_POSTSUPERSCRIPT ⊆ caligraphic_H with {ϕ~+,1n,,ϕ~+,kn}superscriptsubscript~italic-ϕ1𝑛superscriptsubscript~italic-ϕ𝑘𝑛\{\widetilde{\phi}_{+,1}^{n},\dots,\widetilde{\phi}_{+,k}^{n}\}{ over~ start_ARG italic_ϕ end_ARG start_POSTSUBSCRIPT + , 1 end_POSTSUBSCRIPT start_POSTSUPERSCRIPT italic_n end_POSTSUPERSCRIPT , … , over~ start_ARG italic_ϕ end_ARG start_POSTSUBSCRIPT + , italic_k end_POSTSUBSCRIPT start_POSTSUPERSCRIPT italic_n end_POSTSUPERSCRIPT } an orthonormal basis of +nsuperscriptsubscript𝑛\mathcal{H}_{+}^{n}caligraphic_H start_POSTSUBSCRIPT + end_POSTSUBSCRIPT start_POSTSUPERSCRIPT italic_n end_POSTSUPERSCRIPT for sufficiently large n𝑛nitalic_n. Induction in \ellroman_ℓ shows that also ϕ~+,nϕ+,superscriptsubscript~italic-ϕ𝑛subscriptitalic-ϕ\widetilde{\phi}_{+,\ell}^{n}\to\phi_{+,\ell}over~ start_ARG italic_ϕ end_ARG start_POSTSUBSCRIPT + , roman_ℓ end_POSTSUBSCRIPT start_POSTSUPERSCRIPT italic_n end_POSTSUPERSCRIPT → italic_ϕ start_POSTSUBSCRIPT + , roman_ℓ end_POSTSUBSCRIPT for all \ellroman_ℓ. Consequently,

[+n]ψn:=ψ=1kϕ~+,n,ψϕ~+,nψcontainssuperscriptdelimited-[]superscriptsubscript𝑛perpendicular-tosubscript𝜓𝑛assign𝜓superscriptsubscript1𝑘superscriptsubscript~italic-ϕ𝑛𝜓superscriptsubscript~italic-ϕ𝑛𝜓[\mathcal{H}_{+}^{n}]^{\perp}\ni\psi_{n}:=\psi-\sum_{\ell=1}^{k}\langle% \widetilde{\phi}_{+,\ell}^{n}\;,\psi\rangle\widetilde{\phi}_{+,\ell}^{n}\longrightarrow\psi[ caligraphic_H start_POSTSUBSCRIPT + end_POSTSUBSCRIPT start_POSTSUPERSCRIPT italic_n end_POSTSUPERSCRIPT ] start_POSTSUPERSCRIPT ⟂ end_POSTSUPERSCRIPT ∋ italic_ψ start_POSTSUBSCRIPT italic_n end_POSTSUBSCRIPT := italic_ψ - ∑ start_POSTSUBSCRIPT roman_ℓ = 1 end_POSTSUBSCRIPT start_POSTSUPERSCRIPT italic_k end_POSTSUPERSCRIPT ⟨ over~ start_ARG italic_ϕ end_ARG start_POSTSUBSCRIPT + , roman_ℓ end_POSTSUBSCRIPT start_POSTSUPERSCRIPT italic_n end_POSTSUPERSCRIPT , italic_ψ ⟩ over~ start_ARG italic_ϕ end_ARG start_POSTSUBSCRIPT + , roman_ℓ end_POSTSUBSCRIPT start_POSTSUPERSCRIPT italic_n end_POSTSUPERSCRIPT ⟶ italic_ψ

and, since Unψn=0=Uψsubscript𝑈𝑛subscript𝜓𝑛0𝑈𝜓U_{n}\psi_{n}=0=U\psiitalic_U start_POSTSUBSCRIPT italic_n end_POSTSUBSCRIPT italic_ψ start_POSTSUBSCRIPT italic_n end_POSTSUBSCRIPT = 0 = italic_U italic_ψ for large n𝑛nitalic_n, a final application of (4) proves that UnψUψsubscript𝑈𝑛𝜓𝑈𝜓U_{n}\psi\to U\psiitalic_U start_POSTSUBSCRIPT italic_n end_POSTSUBSCRIPT italic_ψ → italic_U italic_ψ. \square

Remark.

Lemma Lemma is actually the main reason why we assume that the deficiency indices of the Ansubscript𝐴𝑛A_{n}italic_A start_POSTSUBSCRIPT italic_n end_POSTSUBSCRIPT’s are finite, since then we can simply restate the condition (3) as strong convergence of the Unsubscript𝑈𝑛U_{n}italic_U start_POSTSUBSCRIPT italic_n end_POSTSUBSCRIPT’s – which is exactly the kind of formulation we seek. If one, in the case of infinite deficiency indices, replaces "UnUsubscript𝑈𝑛𝑈U_{n}\to Uitalic_U start_POSTSUBSCRIPT italic_n end_POSTSUBSCRIPT → italic_U strongly" with (3) then the remaining results of this note in Theorem Theorem and Corollaries Corollary and Corollary indeed remain valid. One can realize that these conditions are truly different in the infinite case by taking the Unsubscript𝑈𝑛U_{n}italic_U start_POSTSUBSCRIPT italic_n end_POSTSUBSCRIPT’s and the U𝑈Uitalic_U to be projections and recalling the content of Remark Remark. \square

While this description of strong convergence of the Unsubscript𝑈𝑛U_{n}italic_U start_POSTSUBSCRIPT italic_n end_POSTSUBSCRIPT’s not at first sight simplifies things, the fact that it is so closely related to the definition of strong graph convergence will help us apply our theory from Section 2 via Theorem Theorem. With this, we are now in a position to state and prove the main theoretical statement of this note.

Theorem.

Let {An}n=1superscriptsubscriptsubscript𝐴𝑛𝑛1\{A_{n}\}_{n=1}^{\infty}{ italic_A start_POSTSUBSCRIPT italic_n end_POSTSUBSCRIPT } start_POSTSUBSCRIPT italic_n = 1 end_POSTSUBSCRIPT start_POSTSUPERSCRIPT ∞ end_POSTSUPERSCRIPT, {Bn}n=1superscriptsubscriptsubscript𝐵𝑛𝑛1\{B_{n}\}_{n=1}^{\infty}{ italic_B start_POSTSUBSCRIPT italic_n end_POSTSUBSCRIPT } start_POSTSUBSCRIPT italic_n = 1 end_POSTSUBSCRIPT start_POSTSUPERSCRIPT ∞ end_POSTSUPERSCRIPT, {Un}n=1superscriptsubscriptsubscript𝑈𝑛𝑛1\{U_{n}\}_{n=1}^{\infty}{ italic_U start_POSTSUBSCRIPT italic_n end_POSTSUBSCRIPT } start_POSTSUBSCRIPT italic_n = 1 end_POSTSUBSCRIPT start_POSTSUPERSCRIPT ∞ end_POSTSUPERSCRIPT, A𝐴Aitalic_A, B𝐵Bitalic_B and U𝑈Uitalic_U be as in the beginning of Section 1. Let ΓsubscriptΓ\Gamma_{\infty}roman_Γ start_POSTSUBSCRIPT ∞ end_POSTSUBSCRIPT be the strong limit of {Gr(An)}n=1superscriptsubscriptGrsubscript𝐴𝑛𝑛1\{\operatorname{Gr}(A_{n})\}_{n=1}^{\infty}{ roman_Gr ( italic_A start_POSTSUBSCRIPT italic_n end_POSTSUBSCRIPT ) } start_POSTSUBSCRIPT italic_n = 1 end_POSTSUBSCRIPT start_POSTSUPERSCRIPT ∞ end_POSTSUPERSCRIPT, and denote by Pnsubscript𝑃𝑛P_{n}italic_P start_POSTSUBSCRIPT italic_n end_POSTSUBSCRIPT and P𝑃Pitalic_P the orthogonal projections in direct-sum\mathcal{H}\oplus\mathcal{H}caligraphic_H ⊕ caligraphic_H onto Gr(An)Grsubscript𝐴𝑛\operatorname{Gr}(A_{n})roman_Gr ( italic_A start_POSTSUBSCRIPT italic_n end_POSTSUBSCRIPT ) and Gr(A)Gr𝐴\operatorname{Gr}(A)roman_Gr ( italic_A ) respectively. Then the following holds:

  • (a)

    If Gr(A)ΓGr𝐴subscriptΓ\operatorname{Gr}(A)\subseteq\Gamma_{\infty}roman_Gr ( italic_A ) ⊆ roman_Γ start_POSTSUBSCRIPT ∞ end_POSTSUBSCRIPT and UnUsubscript𝑈𝑛𝑈U_{n}\to Uitalic_U start_POSTSUBSCRIPT italic_n end_POSTSUBSCRIPT → italic_U strongly then BnBsubscript𝐵𝑛𝐵B_{n}\to Bitalic_B start_POSTSUBSCRIPT italic_n end_POSTSUBSCRIPT → italic_B in the strong resolvent sense and PnPsubscript𝑃𝑛𝑃P_{n}\to Pitalic_P start_POSTSUBSCRIPT italic_n end_POSTSUBSCRIPT → italic_P strongly.

  • (b)

    If BnBsubscript𝐵𝑛𝐵B_{n}\to Bitalic_B start_POSTSUBSCRIPT italic_n end_POSTSUBSCRIPT → italic_B in the strong resolvent sense and PnPsubscript𝑃𝑛𝑃P_{n}\to Pitalic_P start_POSTSUBSCRIPT italic_n end_POSTSUBSCRIPT → italic_P strongly then UnUsubscript𝑈𝑛𝑈U_{n}\to Uitalic_U start_POSTSUBSCRIPT italic_n end_POSTSUBSCRIPT → italic_U strongly.

\square

Proof.

(a): Recall that, cf. [1] X.1,

Gr(B)=Gr(A){(ϕ++Uϕ+,iϕ+iUϕ+)|ϕ++},Gr𝐵direct-sumGr𝐴|subscriptitalic-ϕ𝑈subscriptitalic-ϕ𝑖subscriptitalic-ϕ𝑖𝑈subscriptitalic-ϕsubscriptitalic-ϕsubscript\operatorname{Gr}(B)=\operatorname{Gr}(A)\oplus\{(\phi_{+}+U\phi_{+},i\phi_{+}% -iU\phi_{+})\mathrel{|}\phi_{+}\in\mathcal{H}_{+}\},roman_Gr ( italic_B ) = roman_Gr ( italic_A ) ⊕ { ( italic_ϕ start_POSTSUBSCRIPT + end_POSTSUBSCRIPT + italic_U italic_ϕ start_POSTSUBSCRIPT + end_POSTSUBSCRIPT , italic_i italic_ϕ start_POSTSUBSCRIPT + end_POSTSUBSCRIPT - italic_i italic_U italic_ϕ start_POSTSUBSCRIPT + end_POSTSUBSCRIPT ) | italic_ϕ start_POSTSUBSCRIPT + end_POSTSUBSCRIPT ∈ caligraphic_H start_POSTSUBSCRIPT + end_POSTSUBSCRIPT } ,

where the sum is orthogonal, from which the k𝑘kitalic_k-dimensional orthogonal complement of Gr(A)Gr𝐴\operatorname{Gr}(A)roman_Gr ( italic_A ) in Gr(B)Gr𝐵\operatorname{Gr}(B)roman_Gr ( italic_B ) is apparent. Applying Lemma Lemma we can for any ϕ++subscriptitalic-ϕsubscript\phi_{+}\in\mathcal{H}_{+}italic_ϕ start_POSTSUBSCRIPT + end_POSTSUBSCRIPT ∈ caligraphic_H start_POSTSUBSCRIPT + end_POSTSUBSCRIPT find {ϕ+n}n=1superscriptsubscriptsuperscriptsubscriptitalic-ϕ𝑛𝑛1\{\phi_{+}^{n}\}_{n=1}^{\infty}\subseteq\mathcal{H}{ italic_ϕ start_POSTSUBSCRIPT + end_POSTSUBSCRIPT start_POSTSUPERSCRIPT italic_n end_POSTSUPERSCRIPT } start_POSTSUBSCRIPT italic_n = 1 end_POSTSUBSCRIPT start_POSTSUPERSCRIPT ∞ end_POSTSUPERSCRIPT ⊆ caligraphic_H so that ϕ+n+nsuperscriptsubscriptitalic-ϕ𝑛superscriptsubscript𝑛\phi_{+}^{n}\in\mathcal{H}_{+}^{n}italic_ϕ start_POSTSUBSCRIPT + end_POSTSUBSCRIPT start_POSTSUPERSCRIPT italic_n end_POSTSUPERSCRIPT ∈ caligraphic_H start_POSTSUBSCRIPT + end_POSTSUBSCRIPT start_POSTSUPERSCRIPT italic_n end_POSTSUPERSCRIPT for all n𝑛nitalic_n and

(ϕ+n+Unϕ+n,iϕ+niUnϕ+n)(ϕ++Uϕ+,iϕ+iUϕ+).superscriptsubscriptitalic-ϕ𝑛subscript𝑈𝑛superscriptsubscriptitalic-ϕ𝑛𝑖superscriptsubscriptitalic-ϕ𝑛𝑖subscript𝑈𝑛superscriptsubscriptitalic-ϕ𝑛subscriptitalic-ϕ𝑈subscriptitalic-ϕ𝑖subscriptitalic-ϕ𝑖𝑈subscriptitalic-ϕ(\phi_{+}^{n}+U_{n}\phi_{+}^{n},i\phi_{+}^{n}-iU_{n}\phi_{+}^{n})% \longrightarrow(\phi_{+}+U\phi_{+},i\phi_{+}-iU\phi_{+}).( italic_ϕ start_POSTSUBSCRIPT + end_POSTSUBSCRIPT start_POSTSUPERSCRIPT italic_n end_POSTSUPERSCRIPT + italic_U start_POSTSUBSCRIPT italic_n end_POSTSUBSCRIPT italic_ϕ start_POSTSUBSCRIPT + end_POSTSUBSCRIPT start_POSTSUPERSCRIPT italic_n end_POSTSUPERSCRIPT , italic_i italic_ϕ start_POSTSUBSCRIPT + end_POSTSUBSCRIPT start_POSTSUPERSCRIPT italic_n end_POSTSUPERSCRIPT - italic_i italic_U start_POSTSUBSCRIPT italic_n end_POSTSUBSCRIPT italic_ϕ start_POSTSUBSCRIPT + end_POSTSUBSCRIPT start_POSTSUPERSCRIPT italic_n end_POSTSUPERSCRIPT ) ⟶ ( italic_ϕ start_POSTSUBSCRIPT + end_POSTSUBSCRIPT + italic_U italic_ϕ start_POSTSUBSCRIPT + end_POSTSUBSCRIPT , italic_i italic_ϕ start_POSTSUBSCRIPT + end_POSTSUBSCRIPT - italic_i italic_U italic_ϕ start_POSTSUBSCRIPT + end_POSTSUBSCRIPT ) .

Hence, Corollary Corollary implies BnBsubscript𝐵𝑛𝐵B_{n}\to Bitalic_B start_POSTSUBSCRIPT italic_n end_POSTSUBSCRIPT → italic_B in the strong resolvent sense. Likewise we have also (cf. [1] X.1)

Gr(A*)=Gr(A){(ϕ+,iϕ+)|ϕ++}{(Uϕ+,iUϕ+)|ϕ++}Grsuperscript𝐴direct-sumGr𝐴|subscriptitalic-ϕ𝑖subscriptitalic-ϕsubscriptitalic-ϕsubscript|𝑈subscriptitalic-ϕ𝑖𝑈subscriptitalic-ϕsubscriptitalic-ϕsubscript\operatorname{Gr}(A^{*})=\operatorname{Gr}(A)\oplus\{(\phi_{+},i\phi_{+})% \mathrel{|}\phi_{+}\in\mathcal{H}_{+}\}\oplus\{(U\phi_{+},-iU\phi_{+})\mathrel% {|}\phi_{+}\in\mathcal{H}_{+}\}roman_Gr ( italic_A start_POSTSUPERSCRIPT * end_POSTSUPERSCRIPT ) = roman_Gr ( italic_A ) ⊕ { ( italic_ϕ start_POSTSUBSCRIPT + end_POSTSUBSCRIPT , italic_i italic_ϕ start_POSTSUBSCRIPT + end_POSTSUBSCRIPT ) | italic_ϕ start_POSTSUBSCRIPT + end_POSTSUBSCRIPT ∈ caligraphic_H start_POSTSUBSCRIPT + end_POSTSUBSCRIPT } ⊕ { ( italic_U italic_ϕ start_POSTSUBSCRIPT + end_POSTSUBSCRIPT , - italic_i italic_U italic_ϕ start_POSTSUBSCRIPT + end_POSTSUBSCRIPT ) | italic_ϕ start_POSTSUBSCRIPT + end_POSTSUBSCRIPT ∈ caligraphic_H start_POSTSUBSCRIPT + end_POSTSUBSCRIPT }

and a similar application of Lemma Lemma tells us that Gr(A*)Γ*Grsuperscript𝐴superscriptsubscriptΓ\operatorname{Gr}(A^{*})\subseteq\Gamma_{\infty}^{*}roman_Gr ( italic_A start_POSTSUPERSCRIPT * end_POSTSUPERSCRIPT ) ⊆ roman_Γ start_POSTSUBSCRIPT ∞ end_POSTSUBSCRIPT start_POSTSUPERSCRIPT * end_POSTSUPERSCRIPT (the strong limit of {Gr(An*)}n=1superscriptsubscriptGrsuperscriptsubscript𝐴𝑛𝑛1\{\operatorname{Gr}(A_{n}^{*})\}_{n=1}^{\infty}{ roman_Gr ( italic_A start_POSTSUBSCRIPT italic_n end_POSTSUBSCRIPT start_POSTSUPERSCRIPT * end_POSTSUPERSCRIPT ) } start_POSTSUBSCRIPT italic_n = 1 end_POSTSUBSCRIPT start_POSTSUPERSCRIPT ∞ end_POSTSUPERSCRIPT). By invoking Proposition 2 we get thus additionally PnPsubscript𝑃𝑛𝑃P_{n}\to Pitalic_P start_POSTSUBSCRIPT italic_n end_POSTSUBSCRIPT → italic_P strongly.

(b): We note that by Theorem Theorem (and using the notation herein) we have QnQsubscript𝑄𝑛𝑄Q_{n}\to Qitalic_Q start_POSTSUBSCRIPT italic_n end_POSTSUBSCRIPT → italic_Q strongly, and consequently QnPnQPsubscript𝑄𝑛subscript𝑃𝑛𝑄𝑃Q_{n}-P_{n}\to Q-Pitalic_Q start_POSTSUBSCRIPT italic_n end_POSTSUBSCRIPT - italic_P start_POSTSUBSCRIPT italic_n end_POSTSUBSCRIPT → italic_Q - italic_P strongly. Now QnPnsubscript𝑄𝑛subscript𝑃𝑛Q_{n}-P_{n}italic_Q start_POSTSUBSCRIPT italic_n end_POSTSUBSCRIPT - italic_P start_POSTSUBSCRIPT italic_n end_POSTSUBSCRIPT is the orthogonal projection onto the orthogonal complement of Gr(An)Grsubscript𝐴𝑛\operatorname{Gr}(A_{n})roman_Gr ( italic_A start_POSTSUBSCRIPT italic_n end_POSTSUBSCRIPT ) inside Gr(Bn)Grsubscript𝐵𝑛\operatorname{Gr}(B_{n})roman_Gr ( italic_B start_POSTSUBSCRIPT italic_n end_POSTSUBSCRIPT ) and similarly for QP𝑄𝑃Q-Pitalic_Q - italic_P. But we have just seen in the proof of (a) that these are exactly

{(ϕ+n+Unϕ+n,iϕ+niUnϕ+n)|ϕ+n+n}and{(ϕ++Uϕ+,iϕ+iUϕ+)|ϕ++}|superscriptsubscriptitalic-ϕ𝑛subscript𝑈𝑛superscriptsubscriptitalic-ϕ𝑛𝑖superscriptsubscriptitalic-ϕ𝑛𝑖subscript𝑈𝑛superscriptsubscriptitalic-ϕ𝑛superscriptsubscriptitalic-ϕ𝑛superscriptsubscript𝑛and|subscriptitalic-ϕ𝑈subscriptitalic-ϕ𝑖subscriptitalic-ϕ𝑖𝑈subscriptitalic-ϕsubscriptitalic-ϕsubscript\{(\phi_{+}^{n}+U_{n}\phi_{+}^{n},i\phi_{+}^{n}-iU_{n}\phi_{+}^{n})\mathrel{|}% \phi_{+}^{n}\in\mathcal{H}_{+}^{n}\}\quad\text{and}\quad\{(\phi_{+}+U\phi_{+},% i\phi_{+}-iU\phi_{+})\mathrel{|}\phi_{+}\in\mathcal{H}_{+}\}{ ( italic_ϕ start_POSTSUBSCRIPT + end_POSTSUBSCRIPT start_POSTSUPERSCRIPT italic_n end_POSTSUPERSCRIPT + italic_U start_POSTSUBSCRIPT italic_n end_POSTSUBSCRIPT italic_ϕ start_POSTSUBSCRIPT + end_POSTSUBSCRIPT start_POSTSUPERSCRIPT italic_n end_POSTSUPERSCRIPT , italic_i italic_ϕ start_POSTSUBSCRIPT + end_POSTSUBSCRIPT start_POSTSUPERSCRIPT italic_n end_POSTSUPERSCRIPT - italic_i italic_U start_POSTSUBSCRIPT italic_n end_POSTSUBSCRIPT italic_ϕ start_POSTSUBSCRIPT + end_POSTSUBSCRIPT start_POSTSUPERSCRIPT italic_n end_POSTSUPERSCRIPT ) | italic_ϕ start_POSTSUBSCRIPT + end_POSTSUBSCRIPT start_POSTSUPERSCRIPT italic_n end_POSTSUPERSCRIPT ∈ caligraphic_H start_POSTSUBSCRIPT + end_POSTSUBSCRIPT start_POSTSUPERSCRIPT italic_n end_POSTSUPERSCRIPT } and { ( italic_ϕ start_POSTSUBSCRIPT + end_POSTSUBSCRIPT + italic_U italic_ϕ start_POSTSUBSCRIPT + end_POSTSUBSCRIPT , italic_i italic_ϕ start_POSTSUBSCRIPT + end_POSTSUBSCRIPT - italic_i italic_U italic_ϕ start_POSTSUBSCRIPT + end_POSTSUBSCRIPT ) | italic_ϕ start_POSTSUBSCRIPT + end_POSTSUBSCRIPT ∈ caligraphic_H start_POSTSUBSCRIPT + end_POSTSUBSCRIPT }

respectively. Hence, Lemma Lemma(b) tells us that for each ϕ++subscriptitalic-ϕsubscript\phi_{+}\in\mathcal{H}_{+}italic_ϕ start_POSTSUBSCRIPT + end_POSTSUBSCRIPT ∈ caligraphic_H start_POSTSUBSCRIPT + end_POSTSUBSCRIPT there exists a sequence {ϕ+n}n=1superscriptsubscriptsuperscriptsubscriptitalic-ϕ𝑛𝑛1\{\phi_{+}^{n}\}_{n=1}^{\infty}\subseteq\mathcal{H}{ italic_ϕ start_POSTSUBSCRIPT + end_POSTSUBSCRIPT start_POSTSUPERSCRIPT italic_n end_POSTSUPERSCRIPT } start_POSTSUBSCRIPT italic_n = 1 end_POSTSUBSCRIPT start_POSTSUPERSCRIPT ∞ end_POSTSUPERSCRIPT ⊆ caligraphic_H so that ϕ+n+nsuperscriptsubscriptitalic-ϕ𝑛superscriptsubscript𝑛\phi_{+}^{n}\in\mathcal{H}_{+}^{n}italic_ϕ start_POSTSUBSCRIPT + end_POSTSUBSCRIPT start_POSTSUPERSCRIPT italic_n end_POSTSUPERSCRIPT ∈ caligraphic_H start_POSTSUBSCRIPT + end_POSTSUBSCRIPT start_POSTSUPERSCRIPT italic_n end_POSTSUPERSCRIPT for all n𝑛nitalic_n and

(ϕ+n+Unϕ+n,iϕ+niUnϕ+n)(ϕ++Uϕ+,iϕ+iUϕ+).superscriptsubscriptitalic-ϕ𝑛subscript𝑈𝑛superscriptsubscriptitalic-ϕ𝑛𝑖superscriptsubscriptitalic-ϕ𝑛𝑖subscript𝑈𝑛superscriptsubscriptitalic-ϕ𝑛subscriptitalic-ϕ𝑈subscriptitalic-ϕ𝑖subscriptitalic-ϕ𝑖𝑈subscriptitalic-ϕ(\phi_{+}^{n}+U_{n}\phi_{+}^{n},i\phi_{+}^{n}-iU_{n}\phi_{+}^{n})% \longrightarrow(\phi_{+}+U\phi_{+},i\phi_{+}-iU\phi_{+}).( italic_ϕ start_POSTSUBSCRIPT + end_POSTSUBSCRIPT start_POSTSUPERSCRIPT italic_n end_POSTSUPERSCRIPT + italic_U start_POSTSUBSCRIPT italic_n end_POSTSUBSCRIPT italic_ϕ start_POSTSUBSCRIPT + end_POSTSUBSCRIPT start_POSTSUPERSCRIPT italic_n end_POSTSUPERSCRIPT , italic_i italic_ϕ start_POSTSUBSCRIPT + end_POSTSUBSCRIPT start_POSTSUPERSCRIPT italic_n end_POSTSUPERSCRIPT - italic_i italic_U start_POSTSUBSCRIPT italic_n end_POSTSUBSCRIPT italic_ϕ start_POSTSUBSCRIPT + end_POSTSUBSCRIPT start_POSTSUPERSCRIPT italic_n end_POSTSUPERSCRIPT ) ⟶ ( italic_ϕ start_POSTSUBSCRIPT + end_POSTSUBSCRIPT + italic_U italic_ϕ start_POSTSUBSCRIPT + end_POSTSUBSCRIPT , italic_i italic_ϕ start_POSTSUBSCRIPT + end_POSTSUBSCRIPT - italic_i italic_U italic_ϕ start_POSTSUBSCRIPT + end_POSTSUBSCRIPT ) .

By taking linear combinations of the entries we see that for this sequence ϕ+nϕ+superscriptsubscriptitalic-ϕ𝑛subscriptitalic-ϕ\phi_{+}^{n}\to\phi_{+}italic_ϕ start_POSTSUBSCRIPT + end_POSTSUBSCRIPT start_POSTSUPERSCRIPT italic_n end_POSTSUPERSCRIPT → italic_ϕ start_POSTSUBSCRIPT + end_POSTSUBSCRIPT and Unϕ+nUϕ+subscript𝑈𝑛superscriptsubscriptitalic-ϕ𝑛𝑈subscriptitalic-ϕU_{n}\phi_{+}^{n}\to U\phi_{+}italic_U start_POSTSUBSCRIPT italic_n end_POSTSUBSCRIPT italic_ϕ start_POSTSUBSCRIPT + end_POSTSUBSCRIPT start_POSTSUPERSCRIPT italic_n end_POSTSUPERSCRIPT → italic_U italic_ϕ start_POSTSUBSCRIPT + end_POSTSUBSCRIPT, and to wrap things up Lemma Lemma yields the claimed strong convergence of the Unsubscript𝑈𝑛U_{n}italic_U start_POSTSUBSCRIPT italic_n end_POSTSUBSCRIPT’s towards U𝑈Uitalic_U as needed. \square

Remark.

We present here a more transparent way of proving BnBsubscript𝐵𝑛𝐵B_{n}\to Bitalic_B start_POSTSUBSCRIPT italic_n end_POSTSUBSCRIPT → italic_B in the strong resolvent sense in Theorem Theorem(a) than the one presented above which in particular avoids the use of Corollary Corollary and hence of Theorem Theorem.

Define the subspace V:={ϕ++Uϕ+|ϕ++}assign𝑉|subscriptitalic-ϕ𝑈subscriptitalic-ϕsubscriptitalic-ϕsubscriptV:=\{\phi_{+}+U\phi_{+}\mathrel{|}\phi_{+}\in\mathcal{H}_{+}\}italic_V := { italic_ϕ start_POSTSUBSCRIPT + end_POSTSUBSCRIPT + italic_U italic_ϕ start_POSTSUBSCRIPT + end_POSTSUBSCRIPT | italic_ϕ start_POSTSUBSCRIPT + end_POSTSUBSCRIPT ∈ caligraphic_H start_POSTSUBSCRIPT + end_POSTSUBSCRIPT } in \mathcal{H}caligraphic_H and write

=R(B+i)=R(A+i)+R(B|V+i).𝑅𝐵𝑖𝑅𝐴𝑖𝑅evaluated-at𝐵𝑉𝑖\mathcal{H}=R(B+i)=R(A+i)+R(B|_{V}+i).caligraphic_H = italic_R ( italic_B + italic_i ) = italic_R ( italic_A + italic_i ) + italic_R ( italic_B | start_POSTSUBSCRIPT italic_V end_POSTSUBSCRIPT + italic_i ) .

Now since we assume Gr(A)ΓGr𝐴subscriptΓ\operatorname{Gr}(A)\subseteq\Gamma_{\infty}roman_Gr ( italic_A ) ⊆ roman_Γ start_POSTSUBSCRIPT ∞ end_POSTSUBSCRIPT the convergence of (Bn+i)1superscriptsubscript𝐵𝑛𝑖1(B_{n}+i)^{-1}( italic_B start_POSTSUBSCRIPT italic_n end_POSTSUBSCRIPT + italic_i ) start_POSTSUPERSCRIPT - 1 end_POSTSUPERSCRIPT towards (B+i)1superscript𝐵𝑖1(B+i)^{-1}( italic_B + italic_i ) start_POSTSUPERSCRIPT - 1 end_POSTSUPERSCRIPT on R(A+i)𝑅𝐴𝑖R(A+i)italic_R ( italic_A + italic_i ) is proved as in (iii)\Rightarrow(i) in Theorem Theorem. Notice then that

(B+i)(ϕ++Uϕ+)=2iϕ+and(Bn+i)(ϕ+n+Unϕ+n)=2iϕ+nformulae-sequence𝐵𝑖subscriptitalic-ϕ𝑈subscriptitalic-ϕ2𝑖subscriptitalic-ϕandsubscript𝐵𝑛𝑖superscriptsubscriptitalic-ϕ𝑛subscript𝑈𝑛superscriptsubscriptitalic-ϕ𝑛2𝑖superscriptsubscriptitalic-ϕ𝑛(B+i)(\phi_{+}+U\phi_{+})=2i\phi_{+}\qquad\text{and}\qquad(B_{n}+i)(\phi_{+}^{% n}+U_{n}\phi_{+}^{n})=2i\phi_{+}^{n}( italic_B + italic_i ) ( italic_ϕ start_POSTSUBSCRIPT + end_POSTSUBSCRIPT + italic_U italic_ϕ start_POSTSUBSCRIPT + end_POSTSUBSCRIPT ) = 2 italic_i italic_ϕ start_POSTSUBSCRIPT + end_POSTSUBSCRIPT and ( italic_B start_POSTSUBSCRIPT italic_n end_POSTSUBSCRIPT + italic_i ) ( italic_ϕ start_POSTSUBSCRIPT + end_POSTSUBSCRIPT start_POSTSUPERSCRIPT italic_n end_POSTSUPERSCRIPT + italic_U start_POSTSUBSCRIPT italic_n end_POSTSUBSCRIPT italic_ϕ start_POSTSUBSCRIPT + end_POSTSUBSCRIPT start_POSTSUPERSCRIPT italic_n end_POSTSUPERSCRIPT ) = 2 italic_i italic_ϕ start_POSTSUBSCRIPT + end_POSTSUBSCRIPT start_POSTSUPERSCRIPT italic_n end_POSTSUPERSCRIPT

for any ϕ++subscriptitalic-ϕsubscript\phi_{+}\in\mathcal{H}_{+}italic_ϕ start_POSTSUBSCRIPT + end_POSTSUBSCRIPT ∈ caligraphic_H start_POSTSUBSCRIPT + end_POSTSUBSCRIPT and ϕ+n+nsuperscriptsubscriptitalic-ϕ𝑛superscriptsubscript𝑛\phi_{+}^{n}\in\mathcal{H}_{+}^{n}italic_ϕ start_POSTSUBSCRIPT + end_POSTSUBSCRIPT start_POSTSUPERSCRIPT italic_n end_POSTSUPERSCRIPT ∈ caligraphic_H start_POSTSUBSCRIPT + end_POSTSUBSCRIPT start_POSTSUPERSCRIPT italic_n end_POSTSUPERSCRIPT. This proves that R(B|V+i)=+𝑅evaluated-at𝐵𝑉𝑖subscriptR(B|_{V}+i)=\mathcal{H}_{+}italic_R ( italic_B | start_POSTSUBSCRIPT italic_V end_POSTSUBSCRIPT + italic_i ) = caligraphic_H start_POSTSUBSCRIPT + end_POSTSUBSCRIPT, and for each ϕ++subscriptitalic-ϕsubscript\phi_{+}\in\mathcal{H}_{+}italic_ϕ start_POSTSUBSCRIPT + end_POSTSUBSCRIPT ∈ caligraphic_H start_POSTSUBSCRIPT + end_POSTSUBSCRIPT we can use Lemma Lemma to find a sequence {ϕ+n}n=1superscriptsubscriptsuperscriptsubscriptitalic-ϕ𝑛𝑛1\{\phi_{+}^{n}\}_{n=1}^{\infty}\subseteq\mathcal{H}{ italic_ϕ start_POSTSUBSCRIPT + end_POSTSUBSCRIPT start_POSTSUPERSCRIPT italic_n end_POSTSUPERSCRIPT } start_POSTSUBSCRIPT italic_n = 1 end_POSTSUBSCRIPT start_POSTSUPERSCRIPT ∞ end_POSTSUPERSCRIPT ⊆ caligraphic_H so that ϕ+n+nsuperscriptsubscriptitalic-ϕ𝑛superscriptsubscript𝑛\phi_{+}^{n}\in\mathcal{H}_{+}^{n}italic_ϕ start_POSTSUBSCRIPT + end_POSTSUBSCRIPT start_POSTSUPERSCRIPT italic_n end_POSTSUPERSCRIPT ∈ caligraphic_H start_POSTSUBSCRIPT + end_POSTSUBSCRIPT start_POSTSUPERSCRIPT italic_n end_POSTSUPERSCRIPT for each n𝑛nitalic_n and

(Bn+i)1ϕ+(B+\displaystyle\lVert(B_{n}+i)^{-1}\phi_{+}-(B+∥ ( italic_B start_POSTSUBSCRIPT italic_n end_POSTSUBSCRIPT + italic_i ) start_POSTSUPERSCRIPT - 1 end_POSTSUPERSCRIPT italic_ϕ start_POSTSUBSCRIPT + end_POSTSUBSCRIPT - ( italic_B + i)1ϕ+\displaystyle i)^{-1}\phi_{+}\rVertitalic_i ) start_POSTSUPERSCRIPT - 1 end_POSTSUPERSCRIPT italic_ϕ start_POSTSUBSCRIPT + end_POSTSUBSCRIPT ∥
(Bn+i)1ϕ+(Bn+i)1ϕ+n+(Bn+i)1ϕ+n(B+i)1ϕ+absentdelimited-∥∥superscriptsubscript𝐵𝑛𝑖1subscriptitalic-ϕsuperscriptsubscript𝐵𝑛𝑖1superscriptsubscriptitalic-ϕ𝑛delimited-∥∥superscriptsubscript𝐵𝑛𝑖1superscriptsubscriptitalic-ϕ𝑛superscript𝐵𝑖1subscriptitalic-ϕ\displaystyle\leq\lVert(B_{n}+i)^{-1}\phi_{+}-(B_{n}+i)^{-1}\phi_{+}^{n}\rVert% +\lVert(B_{n}+i)^{-1}\phi_{+}^{n}-(B+i)^{-1}\phi_{+}\rVert≤ ∥ ( italic_B start_POSTSUBSCRIPT italic_n end_POSTSUBSCRIPT + italic_i ) start_POSTSUPERSCRIPT - 1 end_POSTSUPERSCRIPT italic_ϕ start_POSTSUBSCRIPT + end_POSTSUBSCRIPT - ( italic_B start_POSTSUBSCRIPT italic_n end_POSTSUBSCRIPT + italic_i ) start_POSTSUPERSCRIPT - 1 end_POSTSUPERSCRIPT italic_ϕ start_POSTSUBSCRIPT + end_POSTSUBSCRIPT start_POSTSUPERSCRIPT italic_n end_POSTSUPERSCRIPT ∥ + ∥ ( italic_B start_POSTSUBSCRIPT italic_n end_POSTSUBSCRIPT + italic_i ) start_POSTSUPERSCRIPT - 1 end_POSTSUPERSCRIPT italic_ϕ start_POSTSUBSCRIPT + end_POSTSUBSCRIPT start_POSTSUPERSCRIPT italic_n end_POSTSUPERSCRIPT - ( italic_B + italic_i ) start_POSTSUPERSCRIPT - 1 end_POSTSUPERSCRIPT italic_ϕ start_POSTSUBSCRIPT + end_POSTSUBSCRIPT ∥
ϕ+ϕ+n+12(ϕ+n+Unϕ+n)(ϕ++Uϕ+)0.absentdelimited-∥∥subscriptitalic-ϕsuperscriptsubscriptitalic-ϕ𝑛12delimited-∥∥superscriptsubscriptitalic-ϕ𝑛subscript𝑈𝑛superscriptsubscriptitalic-ϕ𝑛subscriptitalic-ϕ𝑈subscriptitalic-ϕ0\displaystyle\leq\lVert\phi_{+}-\phi_{+}^{n}\rVert+\frac{1}{2}\lVert(\phi_{+}^% {n}+U_{n}\phi_{+}^{n})-(\phi_{+}+U\phi_{+})\rVert\longrightarrow 0.≤ ∥ italic_ϕ start_POSTSUBSCRIPT + end_POSTSUBSCRIPT - italic_ϕ start_POSTSUBSCRIPT + end_POSTSUBSCRIPT start_POSTSUPERSCRIPT italic_n end_POSTSUPERSCRIPT ∥ + divide start_ARG 1 end_ARG start_ARG 2 end_ARG ∥ ( italic_ϕ start_POSTSUBSCRIPT + end_POSTSUBSCRIPT start_POSTSUPERSCRIPT italic_n end_POSTSUPERSCRIPT + italic_U start_POSTSUBSCRIPT italic_n end_POSTSUBSCRIPT italic_ϕ start_POSTSUBSCRIPT + end_POSTSUBSCRIPT start_POSTSUPERSCRIPT italic_n end_POSTSUPERSCRIPT ) - ( italic_ϕ start_POSTSUBSCRIPT + end_POSTSUBSCRIPT + italic_U italic_ϕ start_POSTSUBSCRIPT + end_POSTSUBSCRIPT ) ∥ ⟶ 0 .

\square

We can now use Theorem Theorem to prove various statements of the form (1). Taking AnAsubscript𝐴𝑛𝐴A_{n}\to Aitalic_A start_POSTSUBSCRIPT italic_n end_POSTSUBSCRIPT → italic_A to be in terms of strong convergence of the orthogonal projections onto the graphs, i.e. PnPsubscript𝑃𝑛𝑃P_{n}\to Pitalic_P start_POSTSUBSCRIPT italic_n end_POSTSUBSCRIPT → italic_P strongly, we have also Gr(A)ΓGr𝐴subscriptΓ\operatorname{Gr}(A)\subseteq\Gamma_{\infty}roman_Gr ( italic_A ) ⊆ roman_Γ start_POSTSUBSCRIPT ∞ end_POSTSUBSCRIPT due to Proposition 2, and thus we get a particularly clean statement.

Corollary.

Consider the set-up in Theorem Theorem and suppose PnPsubscript𝑃𝑛𝑃P_{n}\to Pitalic_P start_POSTSUBSCRIPT italic_n end_POSTSUBSCRIPT → italic_P strongly. Then BnBsubscript𝐵𝑛𝐵B_{n}\to Bitalic_B start_POSTSUBSCRIPT italic_n end_POSTSUBSCRIPT → italic_B in the strong resolvent sense if and only if UnUsubscript𝑈𝑛𝑈U_{n}\to Uitalic_U start_POSTSUBSCRIPT italic_n end_POSTSUBSCRIPT → italic_U strongly. \square \square

The downside of Corollary Corollary is, however, that the condition PnPsubscript𝑃𝑛𝑃P_{n}\to Pitalic_P start_POSTSUBSCRIPT italic_n end_POSTSUBSCRIPT → italic_P strongly is often not easy to verify in concrete cases. Another approach is to assume the convergence of the Ansubscript𝐴𝑛A_{n}italic_A start_POSTSUBSCRIPT italic_n end_POSTSUBSCRIPT’s only in the sense that Gr(A)ΓGr𝐴subscriptΓ\operatorname{Gr}(A)\subseteq\Gamma_{\infty}roman_Gr ( italic_A ) ⊆ roman_Γ start_POSTSUBSCRIPT ∞ end_POSTSUBSCRIPT. We note that this is a strictly weaker notion of convergence than strong convergence of the graph projections, so one cannot expect the implications of this assumption to be as strong as the equivalence between strong convergence of the Bnsubscript𝐵𝑛B_{n}italic_B start_POSTSUBSCRIPT italic_n end_POSTSUBSCRIPT’s and of the Unsubscript𝑈𝑛U_{n}italic_U start_POSTSUBSCRIPT italic_n end_POSTSUBSCRIPT’s in Corollary Corollary. Another application of Proposition 2 yields:

Corollary.

Consider the set-up in Theorem Theorem and suppose Gr(A)ΓGr𝐴subscriptΓ\operatorname{Gr}(A)\subseteq\Gamma_{\infty}roman_Gr ( italic_A ) ⊆ roman_Γ start_POSTSUBSCRIPT ∞ end_POSTSUBSCRIPT. Then UnUsubscript𝑈𝑛𝑈U_{n}\to Uitalic_U start_POSTSUBSCRIPT italic_n end_POSTSUBSCRIPT → italic_U strongly if and only if both BnBsubscript𝐵𝑛𝐵B_{n}\to Bitalic_B start_POSTSUBSCRIPT italic_n end_POSTSUBSCRIPT → italic_B in the strong resolvent sense and Gr(A*)Γ*Grsuperscript𝐴superscriptsubscriptΓ\operatorname{Gr}(A^{*})\subseteq\Gamma_{\infty}^{*}roman_Gr ( italic_A start_POSTSUPERSCRIPT * end_POSTSUPERSCRIPT ) ⊆ roman_Γ start_POSTSUBSCRIPT ∞ end_POSTSUBSCRIPT start_POSTSUPERSCRIPT * end_POSTSUPERSCRIPT. \square \square

An obvious question now arises: Is this the best we can do? In particular we can in the light of Corollary Corollary ask whether the condition Gr(A*)Γ*Grsuperscript𝐴superscriptsubscriptΓ\operatorname{Gr}(A^{*})\subseteq\Gamma_{\infty}^{*}roman_Gr ( italic_A start_POSTSUPERSCRIPT * end_POSTSUPERSCRIPT ) ⊆ roman_Γ start_POSTSUBSCRIPT ∞ end_POSTSUBSCRIPT start_POSTSUPERSCRIPT * end_POSTSUPERSCRIPT in Corollary Corollary is actually needed. As a matter of fact it is by the following observations.

Remark.

We do not in general have the result "Suppose Gr(A)ΓGr𝐴subscriptΓ\operatorname{Gr}(A)\subseteq\Gamma_{\infty}roman_Gr ( italic_A ) ⊆ roman_Γ start_POSTSUBSCRIPT ∞ end_POSTSUBSCRIPT. Then UnUsubscript𝑈𝑛𝑈U_{n}\to Uitalic_U start_POSTSUBSCRIPT italic_n end_POSTSUBSCRIPT → italic_U strongly if and only if BnBsubscript𝐵𝑛𝐵B_{n}\to Bitalic_B start_POSTSUBSCRIPT italic_n end_POSTSUBSCRIPT → italic_B in the strong resolvent sense." as the example below shows. Even changing Gr(A)ΓGr𝐴subscriptΓ\operatorname{Gr}(A)\subseteq\Gamma_{\infty}roman_Gr ( italic_A ) ⊆ roman_Γ start_POSTSUBSCRIPT ∞ end_POSTSUBSCRIPT to A=str.gr.limAn𝐴formulae-sequencestrgrlimsubscript𝐴𝑛A=\operatorname{str.gr.lim}A_{n}italic_A = start_OPFUNCTION roman_str . roman_gr . roman_lim end_OPFUNCTION italic_A start_POSTSUBSCRIPT italic_n end_POSTSUBSCRIPT does not make the statement true. The backbone of the example is the extension theory for a well-studied class of operators on L2(3)=superscript𝐿2superscript3L^{2}(\mathbb{R}^{3})=\mathcal{H}italic_L start_POSTSUPERSCRIPT 2 end_POSTSUPERSCRIPT ( blackboard_R start_POSTSUPERSCRIPT 3 end_POSTSUPERSCRIPT ) = caligraphic_H. This is treated in for example [3] I.1.1 to which we refer for the details.

Let {yn}n=13superscriptsubscriptsubscript𝑦𝑛𝑛1superscript3\{y_{n}\}_{n=1}^{\infty}\subseteq\mathbb{R}^{3}{ italic_y start_POSTSUBSCRIPT italic_n end_POSTSUBSCRIPT } start_POSTSUBSCRIPT italic_n = 1 end_POSTSUBSCRIPT start_POSTSUPERSCRIPT ∞ end_POSTSUPERSCRIPT ⊆ blackboard_R start_POSTSUPERSCRIPT 3 end_POSTSUPERSCRIPT be a sequence yet to be specified and define for each n𝑛nitalic_n the operator Ansubscript𝐴𝑛A_{n}italic_A start_POSTSUBSCRIPT italic_n end_POSTSUBSCRIPT to be the closure of ΔΔ-\Delta- roman_Δ on Cc(3\{yn})superscriptsubscript𝐶𝑐\superscript3subscript𝑦𝑛C_{c}^{\infty}(\mathbb{R}^{3}\backslash\{y_{n}\})italic_C start_POSTSUBSCRIPT italic_c end_POSTSUBSCRIPT start_POSTSUPERSCRIPT ∞ end_POSTSUPERSCRIPT ( blackboard_R start_POSTSUPERSCRIPT 3 end_POSTSUPERSCRIPT \ { italic_y start_POSTSUBSCRIPT italic_n end_POSTSUBSCRIPT } ). One can now find the deficiency subspaces

±n=ϕ±n,ϕ±n(x)=ei±i|xyn|4π|xyn|formulae-sequencesuperscriptsubscriptplus-or-minus𝑛superscriptsubscriptitalic-ϕplus-or-minus𝑛superscriptsubscriptitalic-ϕplus-or-minus𝑛𝑥superscript𝑒𝑖plus-or-minus𝑖𝑥subscript𝑦𝑛4π𝑥subscript𝑦𝑛\mathcal{H}_{\pm}^{n}=\mathbb{C}\phi_{\pm}^{n},\qquad\phi_{\pm}^{n}(x)=\frac{e% ^{i\sqrt{\pm i}\lvert x-y_{n}\rvert}}{4\uppi\,\lvert x-y_{n}\rvert}caligraphic_H start_POSTSUBSCRIPT ± end_POSTSUBSCRIPT start_POSTSUPERSCRIPT italic_n end_POSTSUPERSCRIPT = blackboard_C italic_ϕ start_POSTSUBSCRIPT ± end_POSTSUBSCRIPT start_POSTSUPERSCRIPT italic_n end_POSTSUPERSCRIPT , italic_ϕ start_POSTSUBSCRIPT ± end_POSTSUBSCRIPT start_POSTSUPERSCRIPT italic_n end_POSTSUPERSCRIPT ( italic_x ) = divide start_ARG italic_e start_POSTSUPERSCRIPT italic_i square-root start_ARG ± italic_i end_ARG | italic_x - italic_y start_POSTSUBSCRIPT italic_n end_POSTSUBSCRIPT | end_POSTSUPERSCRIPT end_ARG start_ARG 4 roman_π | italic_x - italic_y start_POSTSUBSCRIPT italic_n end_POSTSUBSCRIPT | end_ARG

where ±i>0plus-or-minus𝑖0\Im\sqrt{\pm i}>0roman_ℑ square-root start_ARG ± italic_i end_ARG > 0. Moreover, if one defines a self-adjoint extension Bnsubscript𝐵𝑛B_{n}italic_B start_POSTSUBSCRIPT italic_n end_POSTSUBSCRIPT of Ansubscript𝐴𝑛A_{n}italic_A start_POSTSUBSCRIPT italic_n end_POSTSUBSCRIPT by the unitary map Un:+nϕ+nϕnn:subscript𝑈𝑛containssuperscriptsubscript𝑛superscriptsubscriptitalic-ϕ𝑛maps-tosuperscriptsubscriptitalic-ϕ𝑛superscriptsubscript𝑛U_{n}\colon\mathcal{H}_{+}^{n}\ni\phi_{+}^{n}\mapsto-\phi_{-}^{n}\in\mathcal{H% }_{-}^{n}italic_U start_POSTSUBSCRIPT italic_n end_POSTSUBSCRIPT : caligraphic_H start_POSTSUBSCRIPT + end_POSTSUBSCRIPT start_POSTSUPERSCRIPT italic_n end_POSTSUPERSCRIPT ∋ italic_ϕ start_POSTSUBSCRIPT + end_POSTSUBSCRIPT start_POSTSUPERSCRIPT italic_n end_POSTSUPERSCRIPT ↦ - italic_ϕ start_POSTSUBSCRIPT - end_POSTSUBSCRIPT start_POSTSUPERSCRIPT italic_n end_POSTSUPERSCRIPT ∈ caligraphic_H start_POSTSUBSCRIPT - end_POSTSUBSCRIPT start_POSTSUPERSCRIPT italic_n end_POSTSUPERSCRIPT as in Section 1 then Bn=Bsubscript𝐵𝑛𝐵B_{n}=Bitalic_B start_POSTSUBSCRIPT italic_n end_POSTSUBSCRIPT = italic_B is actually the free Laplacian ΔΔ-\Delta- roman_Δ defined on the Sobolev space H2(3)superscript𝐻2superscript3H^{2}(\mathbb{R}^{3})italic_H start_POSTSUPERSCRIPT 2 end_POSTSUPERSCRIPT ( blackboard_R start_POSTSUPERSCRIPT 3 end_POSTSUPERSCRIPT ) independently of n𝑛nitalic_n. Now we have the orthogonal decomposition

Gr(B)=Gr(An)(ϕ+nϕn,iϕ+n+iϕn)=:Gr(An)vn,\operatorname{Gr}(B)=\operatorname{Gr}(A_{n})\oplus\mathbb{C}(\phi_{+}^{n}-% \phi_{-}^{n},i\phi_{+}^{n}+i\phi_{-}^{n})=:\operatorname{Gr}(A_{n})\oplus% \mathbb{C}v_{n},roman_Gr ( italic_B ) = roman_Gr ( italic_A start_POSTSUBSCRIPT italic_n end_POSTSUBSCRIPT ) ⊕ blackboard_C ( italic_ϕ start_POSTSUBSCRIPT + end_POSTSUBSCRIPT start_POSTSUPERSCRIPT italic_n end_POSTSUPERSCRIPT - italic_ϕ start_POSTSUBSCRIPT - end_POSTSUBSCRIPT start_POSTSUPERSCRIPT italic_n end_POSTSUPERSCRIPT , italic_i italic_ϕ start_POSTSUBSCRIPT + end_POSTSUBSCRIPT start_POSTSUPERSCRIPT italic_n end_POSTSUPERSCRIPT + italic_i italic_ϕ start_POSTSUBSCRIPT - end_POSTSUBSCRIPT start_POSTSUPERSCRIPT italic_n end_POSTSUPERSCRIPT ) = : roman_Gr ( italic_A start_POSTSUBSCRIPT italic_n end_POSTSUBSCRIPT ) ⊕ blackboard_C italic_v start_POSTSUBSCRIPT italic_n end_POSTSUBSCRIPT ,

and consequently Gr(An)Grsubscript𝐴𝑛\operatorname{Gr}(A_{n})roman_Gr ( italic_A start_POSTSUBSCRIPT italic_n end_POSTSUBSCRIPT ) is the orthogonal complement of vnsubscript𝑣𝑛\mathbb{C}v_{n}blackboard_C italic_v start_POSTSUBSCRIPT italic_n end_POSTSUBSCRIPT in Gr(B)Gr𝐵\operatorname{Gr}(B)roman_Gr ( italic_B ) for each n𝑛nitalic_n. Notice now that the vnsubscript𝑣𝑛v_{n}italic_v start_POSTSUBSCRIPT italic_n end_POSTSUBSCRIPT’s depend only on the ynsubscript𝑦𝑛y_{n}italic_y start_POSTSUBSCRIPT italic_n end_POSTSUBSCRIPT’s. Choosing ynsubscript𝑦𝑛y_{n}italic_y start_POSTSUBSCRIPT italic_n end_POSTSUBSCRIPT so that |yn|subscript𝑦𝑛\lvert y_{n}\rvert\to\infty| italic_y start_POSTSUBSCRIPT italic_n end_POSTSUBSCRIPT | → ∞ it is not difficult to realize that the sequences {ϕ±n}n=1superscriptsubscriptsuperscriptsubscriptitalic-ϕplus-or-minus𝑛𝑛1\{\phi_{\pm}^{n}\}_{n=1}^{\infty}{ italic_ϕ start_POSTSUBSCRIPT ± end_POSTSUBSCRIPT start_POSTSUPERSCRIPT italic_n end_POSTSUPERSCRIPT } start_POSTSUBSCRIPT italic_n = 1 end_POSTSUBSCRIPT start_POSTSUPERSCRIPT ∞ end_POSTSUPERSCRIPT converge weakly towards 00 in L2(3)superscript𝐿2superscript3L^{2}(\mathbb{R}^{3})italic_L start_POSTSUPERSCRIPT 2 end_POSTSUPERSCRIPT ( blackboard_R start_POSTSUPERSCRIPT 3 end_POSTSUPERSCRIPT ): This follows from the fact that they are translations of a fixed L2superscript𝐿2L^{2}italic_L start_POSTSUPERSCRIPT 2 end_POSTSUPERSCRIPT-function. With such sequence of ynsubscript𝑦𝑛y_{n}italic_y start_POSTSUBSCRIPT italic_n end_POSTSUBSCRIPT’s we get thus

(ϕ,ψ),vn=ϕ,ϕ+nϕ,ϕn+iψ,ϕ+n+iψ,ϕn0italic-ϕ𝜓subscript𝑣𝑛italic-ϕsuperscriptsubscriptitalic-ϕ𝑛italic-ϕsuperscriptsubscriptitalic-ϕ𝑛𝑖𝜓superscriptsubscriptitalic-ϕ𝑛𝑖𝜓superscriptsubscriptitalic-ϕ𝑛0\langle(\phi,\psi),v_{n}\rangle=\langle\phi,\phi_{+}^{n}\rangle-\langle\phi,% \phi_{-}^{n}\rangle+i\langle\psi,\phi_{+}^{n}\rangle+i\langle\psi,\phi_{-}^{n}% \rangle\longrightarrow 0⟨ ( italic_ϕ , italic_ψ ) , italic_v start_POSTSUBSCRIPT italic_n end_POSTSUBSCRIPT ⟩ = ⟨ italic_ϕ , italic_ϕ start_POSTSUBSCRIPT + end_POSTSUBSCRIPT start_POSTSUPERSCRIPT italic_n end_POSTSUPERSCRIPT ⟩ - ⟨ italic_ϕ , italic_ϕ start_POSTSUBSCRIPT - end_POSTSUBSCRIPT start_POSTSUPERSCRIPT italic_n end_POSTSUPERSCRIPT ⟩ + italic_i ⟨ italic_ψ , italic_ϕ start_POSTSUBSCRIPT + end_POSTSUBSCRIPT start_POSTSUPERSCRIPT italic_n end_POSTSUPERSCRIPT ⟩ + italic_i ⟨ italic_ψ , italic_ϕ start_POSTSUBSCRIPT - end_POSTSUBSCRIPT start_POSTSUPERSCRIPT italic_n end_POSTSUPERSCRIPT ⟩ ⟶ 0

for all (ϕ,ψ)italic-ϕ𝜓direct-sum(\phi,\psi)\in\mathcal{H}\oplus\mathcal{H}( italic_ϕ , italic_ψ ) ∈ caligraphic_H ⊕ caligraphic_H, i.e. vn0subscript𝑣𝑛0v_{n}\to 0italic_v start_POSTSUBSCRIPT italic_n end_POSTSUBSCRIPT → 0 weakly in direct-sum\mathcal{H}\oplus\mathcal{H}caligraphic_H ⊕ caligraphic_H and hence in Gr(B)Gr𝐵\operatorname{Gr}(B)roman_Gr ( italic_B ).

We observe from the above facts that by choosing a sequence of ynsubscript𝑦𝑛y_{n}italic_y start_POSTSUBSCRIPT italic_n end_POSTSUBSCRIPT’s which is a fixed yn=y0subscript𝑦𝑛subscript𝑦0y_{n}=y_{0}italic_y start_POSTSUBSCRIPT italic_n end_POSTSUBSCRIPT = italic_y start_POSTSUBSCRIPT 0 end_POSTSUBSCRIPT for n𝑛nitalic_n odd and with {y2n}n=1superscriptsubscriptsubscript𝑦2𝑛𝑛1\{y_{2n}\}_{n=1}^{\infty}{ italic_y start_POSTSUBSCRIPT 2 italic_n end_POSTSUBSCRIPT } start_POSTSUBSCRIPT italic_n = 1 end_POSTSUBSCRIPT start_POSTSUPERSCRIPT ∞ end_POSTSUPERSCRIPT unbounded we can make the sequence {Gr(An)}n=1superscriptsubscriptGrsubscript𝐴𝑛𝑛1\{\operatorname{Gr}(A_{n})\}_{n=1}^{\infty}{ roman_Gr ( italic_A start_POSTSUBSCRIPT italic_n end_POSTSUBSCRIPT ) } start_POSTSUBSCRIPT italic_n = 1 end_POSTSUBSCRIPT start_POSTSUPERSCRIPT ∞ end_POSTSUPERSCRIPT of subspaces of the Hilbert space Gr(B)Gr𝐵\operatorname{Gr}(B)roman_Gr ( italic_B ) into a sequence like {Vn}n=1superscriptsubscriptsubscript𝑉𝑛𝑛1\{V_{n}\}_{n=1}^{\infty}{ italic_V start_POSTSUBSCRIPT italic_n end_POSTSUBSCRIPT } start_POSTSUBSCRIPT italic_n = 1 end_POSTSUBSCRIPT start_POSTSUPERSCRIPT ∞ end_POSTSUPERSCRIPT in Remark Remark. Consequently, the operator A=A1𝐴subscript𝐴1A=A_{1}italic_A = italic_A start_POSTSUBSCRIPT 1 end_POSTSUBSCRIPT is the strong graph limit of the Ansubscript𝐴𝑛A_{n}italic_A start_POSTSUBSCRIPT italic_n end_POSTSUBSCRIPT’s (and of course BnBsubscript𝐵𝑛𝐵B_{n}\to Bitalic_B start_POSTSUBSCRIPT italic_n end_POSTSUBSCRIPT → italic_B), but the orthogonal projections onto the graphs Gr(An)Grsubscript𝐴𝑛\operatorname{Gr}(A_{n})roman_Gr ( italic_A start_POSTSUBSCRIPT italic_n end_POSTSUBSCRIPT ) do not converge strongly towards the orthogonal projection onto Gr(A)Gr𝐴\operatorname{Gr}(A)roman_Gr ( italic_A ), and hence Theorem Theorem(a) tells us that we cannot have UnUsubscript𝑈𝑛𝑈U_{n}\to Uitalic_U start_POSTSUBSCRIPT italic_n end_POSTSUBSCRIPT → italic_U strongly. Alternatively this can be checked more directly by using Lemma Lemma. \square

We conclude by proving a simple requirement for having Gr(A)ΓGr𝐴subscriptΓ\operatorname{Gr}(A)\subseteq\Gamma_{\infty}roman_Gr ( italic_A ) ⊆ roman_Γ start_POSTSUBSCRIPT ∞ end_POSTSUBSCRIPT, thus providing a procedure for checking the assumptions in Corollary Corollary. Recall that a core for a closed operator A𝐴Aitalic_A is a subspace of D(A)𝐷𝐴D(A)italic_D ( italic_A ) satisfying that the restriction of A𝐴Aitalic_A to this has closure A𝐴Aitalic_A. We obtain now: {pro} Assume that 𝒟𝒟\mathcal{D}caligraphic_D is a common core for A𝐴Aitalic_A and all Ansubscript𝐴𝑛A_{n}italic_A start_POSTSUBSCRIPT italic_n end_POSTSUBSCRIPT’s. If AnϕAϕsubscript𝐴𝑛italic-ϕ𝐴italic-ϕA_{n}\phi\to A\phiitalic_A start_POSTSUBSCRIPT italic_n end_POSTSUBSCRIPT italic_ϕ → italic_A italic_ϕ for all ϕ𝒟italic-ϕ𝒟\phi\in\mathcal{D}italic_ϕ ∈ caligraphic_D then Gr(A)ΓGr𝐴subscriptΓ\operatorname{Gr}(A)\subseteq\Gamma_{\infty}roman_Gr ( italic_A ) ⊆ roman_Γ start_POSTSUBSCRIPT ∞ end_POSTSUBSCRIPT.

Proof.

The assumption tells us that Γ𝒟:={(ϕ,Aϕ)|ϕ𝒟}ΓassignsubscriptΓ𝒟|italic-ϕ𝐴italic-ϕitalic-ϕ𝒟subscriptΓ\Gamma_{\mathcal{D}}:=\{(\phi,A\phi)\mathrel{|}\phi\in\mathcal{D}\}\subseteq% \Gamma_{\infty}roman_Γ start_POSTSUBSCRIPT caligraphic_D end_POSTSUBSCRIPT := { ( italic_ϕ , italic_A italic_ϕ ) | italic_ϕ ∈ caligraphic_D } ⊆ roman_Γ start_POSTSUBSCRIPT ∞ end_POSTSUBSCRIPT. Thus, if we argue that ΓsubscriptΓ\Gamma_{\infty}roman_Γ start_POSTSUBSCRIPT ∞ end_POSTSUBSCRIPT is closed, we have also Gr(A)=Γ𝒟¯ΓGr𝐴¯subscriptΓ𝒟subscriptΓ\operatorname{Gr}(A)=\overline{\Gamma_{\mathcal{D}}}\subseteq\Gamma_{\infty}roman_Gr ( italic_A ) = over¯ start_ARG roman_Γ start_POSTSUBSCRIPT caligraphic_D end_POSTSUBSCRIPT end_ARG ⊆ roman_Γ start_POSTSUBSCRIPT ∞ end_POSTSUBSCRIPT. But closedness is a general property of any strong limit of subspaces by the following argument:

Let {Vn}n=1superscriptsubscriptsubscript𝑉𝑛𝑛1\{V_{n}\}_{n=1}^{\infty}{ italic_V start_POSTSUBSCRIPT italic_n end_POSTSUBSCRIPT } start_POSTSUBSCRIPT italic_n = 1 end_POSTSUBSCRIPT start_POSTSUPERSCRIPT ∞ end_POSTSUPERSCRIPT be any sequence of subspaces of a Hilbert space \mathcal{H}caligraphic_H and denote as usual its strong limit by Vsubscript𝑉V_{\infty}italic_V start_POSTSUBSCRIPT ∞ end_POSTSUBSCRIPT. If we consider an arbitrary convergent sequence {xk}k=1Vsuperscriptsubscriptsubscript𝑥𝑘𝑘1subscript𝑉\{x_{k}\}_{k=1}^{\infty}\subseteq V_{\infty}{ italic_x start_POSTSUBSCRIPT italic_k end_POSTSUBSCRIPT } start_POSTSUBSCRIPT italic_k = 1 end_POSTSUBSCRIPT start_POSTSUPERSCRIPT ∞ end_POSTSUPERSCRIPT ⊆ italic_V start_POSTSUBSCRIPT ∞ end_POSTSUBSCRIPT with limit x0subscript𝑥0x_{0}italic_x start_POSTSUBSCRIPT 0 end_POSTSUBSCRIPT then we need only to find a sequence {x~n}n=1superscriptsubscriptsubscript~𝑥𝑛𝑛1\{\widetilde{x}_{n}\}_{n=1}^{\infty}\subseteq\mathcal{H}{ over~ start_ARG italic_x end_ARG start_POSTSUBSCRIPT italic_n end_POSTSUBSCRIPT } start_POSTSUBSCRIPT italic_n = 1 end_POSTSUBSCRIPT start_POSTSUPERSCRIPT ∞ end_POSTSUPERSCRIPT ⊆ caligraphic_H with x~nVnsubscript~𝑥𝑛subscript𝑉𝑛\widetilde{x}_{n}\in V_{n}over~ start_ARG italic_x end_ARG start_POSTSUBSCRIPT italic_n end_POSTSUBSCRIPT ∈ italic_V start_POSTSUBSCRIPT italic_n end_POSTSUBSCRIPT for all n𝑛nitalic_n such that x~nx0subscript~𝑥𝑛subscript𝑥0\widetilde{x}_{n}\to x_{0}over~ start_ARG italic_x end_ARG start_POSTSUBSCRIPT italic_n end_POSTSUBSCRIPT → italic_x start_POSTSUBSCRIPT 0 end_POSTSUBSCRIPT in order to obtain x0Vsubscript𝑥0subscript𝑉x_{0}\in V_{\infty}italic_x start_POSTSUBSCRIPT 0 end_POSTSUBSCRIPT ∈ italic_V start_POSTSUBSCRIPT ∞ end_POSTSUBSCRIPT and hence prove that Vsubscript𝑉V_{\infty}italic_V start_POSTSUBSCRIPT ∞ end_POSTSUBSCRIPT is closed. We now construct such sequence. Firstly we choose for each k𝑘kitalic_k a sequence {xnk}n=1superscriptsubscriptsuperscriptsubscript𝑥𝑛𝑘𝑛1\{x_{n}^{k}\}_{n=1}^{\infty}{ italic_x start_POSTSUBSCRIPT italic_n end_POSTSUBSCRIPT start_POSTSUPERSCRIPT italic_k end_POSTSUPERSCRIPT } start_POSTSUBSCRIPT italic_n = 1 end_POSTSUBSCRIPT start_POSTSUPERSCRIPT ∞ end_POSTSUPERSCRIPT with xnkVnsuperscriptsubscript𝑥𝑛𝑘subscript𝑉𝑛x_{n}^{k}\in V_{n}italic_x start_POSTSUBSCRIPT italic_n end_POSTSUBSCRIPT start_POSTSUPERSCRIPT italic_k end_POSTSUPERSCRIPT ∈ italic_V start_POSTSUBSCRIPT italic_n end_POSTSUBSCRIPT for all n𝑛nitalic_n and xnkxksuperscriptsubscript𝑥𝑛𝑘subscript𝑥𝑘x_{n}^{k}\to x_{k}italic_x start_POSTSUBSCRIPT italic_n end_POSTSUBSCRIPT start_POSTSUPERSCRIPT italic_k end_POSTSUPERSCRIPT → italic_x start_POSTSUBSCRIPT italic_k end_POSTSUBSCRIPT, and then we take natural numbers N1<N2<N3<subscript𝑁1subscript𝑁2subscript𝑁3N_{1}<N_{2}<N_{3}<\cdotsitalic_N start_POSTSUBSCRIPT 1 end_POSTSUBSCRIPT < italic_N start_POSTSUBSCRIPT 2 end_POSTSUBSCRIPT < italic_N start_POSTSUBSCRIPT 3 end_POSTSUBSCRIPT < ⋯ so that xnkxk<1/kdelimited-∥∥superscriptsubscript𝑥𝑛𝑘subscript𝑥𝑘1𝑘\lVert x_{n}^{k}-x_{k}\rVert<1/k∥ italic_x start_POSTSUBSCRIPT italic_n end_POSTSUBSCRIPT start_POSTSUPERSCRIPT italic_k end_POSTSUPERSCRIPT - italic_x start_POSTSUBSCRIPT italic_k end_POSTSUBSCRIPT ∥ < 1 / italic_k for all nNk𝑛subscript𝑁𝑘n\geq N_{k}italic_n ≥ italic_N start_POSTSUBSCRIPT italic_k end_POSTSUBSCRIPT. Defining x~n:=xn1assignsubscript~𝑥𝑛superscriptsubscript𝑥𝑛1\widetilde{x}_{n}:=x_{n}^{1}over~ start_ARG italic_x end_ARG start_POSTSUBSCRIPT italic_n end_POSTSUBSCRIPT := italic_x start_POSTSUBSCRIPT italic_n end_POSTSUBSCRIPT start_POSTSUPERSCRIPT 1 end_POSTSUPERSCRIPT for n=1,2,,N21𝑛12subscript𝑁21n=1,2,\dots,N_{2}-1italic_n = 1 , 2 , … , italic_N start_POSTSUBSCRIPT 2 end_POSTSUBSCRIPT - 1; x~n:=xn2assignsubscript~𝑥𝑛superscriptsubscript𝑥𝑛2\widetilde{x}_{n}:=x_{n}^{2}over~ start_ARG italic_x end_ARG start_POSTSUBSCRIPT italic_n end_POSTSUBSCRIPT := italic_x start_POSTSUBSCRIPT italic_n end_POSTSUBSCRIPT start_POSTSUPERSCRIPT 2 end_POSTSUPERSCRIPT for n=N2,,N31𝑛subscript𝑁2subscript𝑁31n=N_{2},\dots,N_{3}-1italic_n = italic_N start_POSTSUBSCRIPT 2 end_POSTSUBSCRIPT , … , italic_N start_POSTSUBSCRIPT 3 end_POSTSUBSCRIPT - 1 and generally x~n:=xnkassignsubscript~𝑥𝑛superscriptsubscript𝑥𝑛𝑘\widetilde{x}_{n}:=x_{n}^{k}over~ start_ARG italic_x end_ARG start_POSTSUBSCRIPT italic_n end_POSTSUBSCRIPT := italic_x start_POSTSUBSCRIPT italic_n end_POSTSUBSCRIPT start_POSTSUPERSCRIPT italic_k end_POSTSUPERSCRIPT for n=Nk,,Nk+11𝑛subscript𝑁𝑘subscript𝑁𝑘11n=N_{k},\dots,N_{k+1}-1italic_n = italic_N start_POSTSUBSCRIPT italic_k end_POSTSUBSCRIPT , … , italic_N start_POSTSUBSCRIPT italic_k + 1 end_POSTSUBSCRIPT - 1 one can check using the triangular inequality that this is indeed a sequence with the properties we seek. \square

Example.

To make things even more concrete than requiring pointwise convergence of the Ansubscript𝐴𝑛A_{n}italic_A start_POSTSUBSCRIPT italic_n end_POSTSUBSCRIPT’s on a common core, we can ask what this means for differential operators like those in Example 1. To simplify things let us consider a sequence of Schrödinger operators – that is, the Ansubscript𝐴𝑛A_{n}italic_A start_POSTSUBSCRIPT italic_n end_POSTSUBSCRIPT’s are the closures of Δ+ΦnΔsubscriptΦ𝑛-\Delta+\Phi_{n}- roman_Δ + roman_Φ start_POSTSUBSCRIPT italic_n end_POSTSUBSCRIPT defined on Cc(Ω)L2(Ω)superscriptsubscript𝐶𝑐Ωsuperscript𝐿2ΩC_{c}^{\infty}(\Omega)\subseteq L^{2}(\Omega)italic_C start_POSTSUBSCRIPT italic_c end_POSTSUBSCRIPT start_POSTSUPERSCRIPT ∞ end_POSTSUPERSCRIPT ( roman_Ω ) ⊆ italic_L start_POSTSUPERSCRIPT 2 end_POSTSUPERSCRIPT ( roman_Ω ) for some open set ΩdΩsuperscript𝑑\Omega\subseteq\mathbb{R}^{d}roman_Ω ⊆ blackboard_R start_POSTSUPERSCRIPT italic_d end_POSTSUPERSCRIPT and some potentials ΦnsubscriptΦ𝑛\Phi_{n}roman_Φ start_POSTSUBSCRIPT italic_n end_POSTSUBSCRIPT (say, real-valued and continuous) on this set. Hence, Cc(Ω)superscriptsubscript𝐶𝑐ΩC_{c}^{\infty}(\Omega)italic_C start_POSTSUBSCRIPT italic_c end_POSTSUBSCRIPT start_POSTSUPERSCRIPT ∞ end_POSTSUPERSCRIPT ( roman_Ω ) is a common core for the Ansubscript𝐴𝑛A_{n}italic_A start_POSTSUBSCRIPT italic_n end_POSTSUBSCRIPT’s and also for A=Δ+Φ𝐴ΔΦA=-\Delta+\Phiitalic_A = - roman_Δ + roman_Φ if we define this in the same manner. Now, for any ϕCc(Ω)italic-ϕsuperscriptsubscript𝐶𝑐Ω\phi\in C_{c}^{\infty}(\Omega)italic_ϕ ∈ italic_C start_POSTSUBSCRIPT italic_c end_POSTSUBSCRIPT start_POSTSUPERSCRIPT ∞ end_POSTSUPERSCRIPT ( roman_Ω ),

AnϕAϕ2=ΦnϕΦϕ2=Ω|ϕ|2|ΦnΦ|2𝑑xϕ2suppϕ|ΦnΦ|2𝑑xsuperscriptdelimited-∥∥subscript𝐴𝑛italic-ϕ𝐴italic-ϕ2superscriptdelimited-∥∥subscriptΦ𝑛italic-ϕΦitalic-ϕ2subscriptΩsuperscriptitalic-ϕ2superscriptsubscriptΦ𝑛Φ2differential-d𝑥superscriptsubscriptdelimited-∥∥italic-ϕ2subscriptsuppitalic-ϕsuperscriptsubscriptΦ𝑛Φ2differential-d𝑥\lVert A_{n}\phi-A\phi\rVert^{2}=\lVert\Phi_{n}\phi-\Phi\phi\rVert^{2}=\int_{% \Omega}\lvert\phi\rvert^{2}\lvert\Phi_{n}-\Phi\rvert^{2}\,dx\leq\lVert\phi% \rVert_{\infty}^{2}\int_{\operatorname{supp}\phi}\lvert\Phi_{n}-\Phi\rvert^{2}% \,dx∥ italic_A start_POSTSUBSCRIPT italic_n end_POSTSUBSCRIPT italic_ϕ - italic_A italic_ϕ ∥ start_POSTSUPERSCRIPT 2 end_POSTSUPERSCRIPT = ∥ roman_Φ start_POSTSUBSCRIPT italic_n end_POSTSUBSCRIPT italic_ϕ - roman_Φ italic_ϕ ∥ start_POSTSUPERSCRIPT 2 end_POSTSUPERSCRIPT = ∫ start_POSTSUBSCRIPT roman_Ω end_POSTSUBSCRIPT | italic_ϕ | start_POSTSUPERSCRIPT 2 end_POSTSUPERSCRIPT | roman_Φ start_POSTSUBSCRIPT italic_n end_POSTSUBSCRIPT - roman_Φ | start_POSTSUPERSCRIPT 2 end_POSTSUPERSCRIPT italic_d italic_x ≤ ∥ italic_ϕ ∥ start_POSTSUBSCRIPT ∞ end_POSTSUBSCRIPT start_POSTSUPERSCRIPT 2 end_POSTSUPERSCRIPT ∫ start_POSTSUBSCRIPT roman_supp italic_ϕ end_POSTSUBSCRIPT | roman_Φ start_POSTSUBSCRIPT italic_n end_POSTSUBSCRIPT - roman_Φ | start_POSTSUPERSCRIPT 2 end_POSTSUPERSCRIPT italic_d italic_x

where subscriptdelimited-∥∥\lVert\,\cdot\,\rVert_{\infty}∥ ⋅ ∥ start_POSTSUBSCRIPT ∞ end_POSTSUBSCRIPT is the supremum norm. Now if ΦnΦsubscriptΦ𝑛Φ\Phi_{n}\to\Phiroman_Φ start_POSTSUBSCRIPT italic_n end_POSTSUBSCRIPT → roman_Φ in Lloc2(Ω)subscriptsuperscript𝐿2locΩL^{2}_{\operatorname{loc}}(\Omega)italic_L start_POSTSUPERSCRIPT 2 end_POSTSUPERSCRIPT start_POSTSUBSCRIPT roman_loc end_POSTSUBSCRIPT ( roman_Ω ) then we conclude that AnϕAϕsubscript𝐴𝑛italic-ϕ𝐴italic-ϕA_{n}\phi\to A\phiitalic_A start_POSTSUBSCRIPT italic_n end_POSTSUBSCRIPT italic_ϕ → italic_A italic_ϕ for all ϕCc(Ω)italic-ϕsuperscriptsubscript𝐶𝑐Ω\phi\in C_{c}^{\infty}(\Omega)italic_ϕ ∈ italic_C start_POSTSUBSCRIPT italic_c end_POSTSUBSCRIPT start_POSTSUPERSCRIPT ∞ end_POSTSUPERSCRIPT ( roman_Ω ). If, on the other hand, we assume the latter, then we see that ΦnΦsubscriptΦ𝑛Φ\Phi_{n}\to\Phiroman_Φ start_POSTSUBSCRIPT italic_n end_POSTSUBSCRIPT → roman_Φ in L2(K)superscript𝐿2𝐾L^{2}(K)italic_L start_POSTSUPERSCRIPT 2 end_POSTSUPERSCRIPT ( italic_K ) for any compact subset KΩ𝐾ΩK\subseteq\Omegaitalic_K ⊆ roman_Ω by choosing ϕ1italic-ϕ1\phi\equiv 1italic_ϕ ≡ 1 on K𝐾Kitalic_K, i.e. we get ΦnΦsubscriptΦ𝑛Φ\Phi_{n}\to\Phiroman_Φ start_POSTSUBSCRIPT italic_n end_POSTSUBSCRIPT → roman_Φ in Lloc2(Ω)subscriptsuperscript𝐿2locΩL^{2}_{\operatorname{loc}}(\Omega)italic_L start_POSTSUPERSCRIPT 2 end_POSTSUPERSCRIPT start_POSTSUBSCRIPT roman_loc end_POSTSUBSCRIPT ( roman_Ω ). Being able to consider only local L2superscript𝐿2L^{2}italic_L start_POSTSUPERSCRIPT 2 end_POSTSUPERSCRIPT-convergence is often desirable if one deals for example with potentials with singularities. \square

Acknowledgements

This work was supported in parts by the VILLUM Foundation grant no. 10059. I would like to thank my PhD advisor Jan Philip Solovej for initiating this cute little project as well as for his always committed and insightful guidance. Also, I thank Johannes Agerskov for proofreading and for his useful suggestions.

Comments

This version of the article has been accepted for publication, after peer review but is not the Version of Record and does not reflect post-acceptance improvements, or any corrections. The Version of Record is available online at https://doi.org/10.1007/s00023-023-01397-9.

References

  • Reed and Simon [1975] Michael Reed and Barry Simon. Methods of Modern Mathematical Physics. II Fourier Analysis, Self-adjointness. Academic Press, 1975.
  • Reed and Simon [1980] Michael Reed and Barry Simon. Methods of Modern Mathematical Physics. I Functional Analysis. Academic Press, 2 edition, 1980.
  • Albeverio et al. [1988] Sergio Albeverio, Friedrich Gesztesy, Raphael Høegh-Krohn, and Helge Holden. Solvable Models in Quantum Mechanics. Springer-Verlag, 1988.
  • Grubb [2009] Gerd Grubb. Distributions and Operators. Springer, 2009.
  • Portmann et al. [2018] Fabian Portmann, Jérémy Sok, and Jan Philip Solovej. Self-adjointness and spectral properties of dirac operators with magnetic links. Journal de Mathématiques Pures et Appliquées, 119:114–157, 2018. 10.1016/j.matpur.2017.10.010.